UWorld Flashcards

1
Q

Erythematous, hot, shiny, swollen area in skin with irregular borders - hx of DM - dx?

A

Cellulitis - infxn involving deep dermis and superficial fat (vs. erysipelas which involves superficial dermis and lymphatics)

How well did you know this?
1
Not at all
2
3
4
5
Perfectly
2
Q

Can you use lidocaine in cases of cellulitis? Why?

A

Areas of infections tend to be slightly acidic and local anesthetics tend to be basic in nature, thus easily neutralized and become ineffective. If the cellulitis progressed to abscess formation, local anesthetic should still be used to anesthetize the epidermis for draining purposes (not pain control)

How well did you know this?
1
Not at all
2
3
4
5
Perfectly
3
Q

Locally aggressive benign tumor arising from fibroplastic elements in muscle or fascial planes w/ very low potential for metastasis or proliferation = dx? Complications? Trmt?

A

Desmoid tumor. Deeply seeded painless mostly in the trunk/extremity, intraabdominal bowel and mesentery, and abdominal wall. Complications = intestinal obstruction and bowel ischemia. HIGH RATE OF RECURRENCE!

Work-up includes CT/MRI if large, symptomatic, or worrisome for histologic dx. Trmt = surgery or radiation for non-surgical candidates

How well did you know this?
1
Not at all
2
3
4
5
Perfectly
4
Q

Firm hyper pigmented nodule MC on lower extremities seen after trauma = dx?

A

Dermatofibroma

How well did you know this?
1
Not at all
2
3
4
5
Perfectly
5
Q

Results from normal epidermal keratin becoming lodged in the dermis = dx?

A

Epidermoid cyst

How well did you know this?
1
Not at all
2
3
4
5
Perfectly
6
Q

Asymptomatic skin lesion and benign collection of fat cells = dx? Recurrence risk?

A

Lipoma. Low risk

How well did you know this?
1
Not at all
2
3
4
5
Perfectly
7
Q

Capillary proliferation after trauma and is usually dome-shaped papule with recurrent bleeding; seen in pregnant women = ?

A

Pyogenic Granuloma

How well did you know this?
1
Not at all
2
3
4
5
Perfectly
8
Q

SIRS Criteria? Desired CVP in suspected hypoperfusion from sepsis?

A

> 38.5 T, HR > 90, RR > 20, WBC > 12,000 (need 2/4 criteria)

CVP 8-12 via fluid resuscitation (central venous line if necessary). Vasopressors may be needed if persistently hypotensive (Norepinephrine; Dobutamine = more inotropic in suspected poor CO)

How well did you know this?
1
Not at all
2
3
4
5
Perfectly
9
Q

Cushinoid appearance clues you in to chronic use of what? How does this affect management in sick patients?

A

Chronic use of steroids; could lead to HPA suppression. Low cortisol level (even in non-chronic corticosteroid users) can be dangerous in shock situations. Should begin tapering dose after 8hrs of presentation for those in shock without chronic use of steroids w/ SBP

How well did you know this?
1
Not at all
2
3
4
5
Perfectly
10
Q

ADHD course from childhood to adulthood? Increased risk of substance use/abuse w/ stimulant therapy?

A

Hyperactive physical symptoms tends to subside into adolescence while one to two-thirds of children dx’d with ADHD will experience persistent ADHD into adulthood,

No data supporting increased substance use/abuse with stimulants.

How well did you know this?
1
Not at all
2
3
4
5
Perfectly
11
Q

Cardiac exam signs (3) indicating severe AS?

A

Soft/single S2, delayed and diminished carotid pulse “parvus and tardus”, loud and late systolic murmur

How well did you know this?
1
Not at all
2
3
4
5
Perfectly
12
Q

Work-up of Thyroid Nodule: ?

A

First: Clinical Eval (fam hx, past radiation, cervical lymphadenopathy, compressing sympt, symptomatic) / TSH Level (NL or elevated TSH) / US (hypoechoic, microcalcifications, internal vascularity)

No RF’s and Low TSH - I-123 Scintigraphy > hot (increased uptake) can treat as hyperthyroid, cold (low uptake) = concerning, get FNA.

RF’s - get FNA

No RF’s and NL or Elevated TSH - FNA, treat findings

How well did you know this?
1
Not at all
2
3
4
5
Perfectly
13
Q

What should your work-up include with someone who has medullary thyroid cancer? What’s included in each of these categories?

A

Serum calcitonin and carcinoembryonic antigen, neck U/S (regional mets?), genetic testing for germline RET mutations, and eval for coexisting tumors (hyperparathyroidism, PCCs)

MEN 1 - (PPP) pituitary tumor, primary hyperparathyroidism, enteropancreatic tumors
MEN 2A - (MPP) MTC, parathyroid hyperplasia, pheochromocytoma [Familial Medullary Thyroid Cancer = subset of 2A w/ only MTC]
MEN 2B - (MMP) MTC, pheochromocytoma, mucosal/intestinal neuromas, marfinoid habitus

How well did you know this?
1
Not at all
2
3
4
5
Perfectly
14
Q

In the setting of a new pleural effusion w/ a patient being treated for TB, what should you order?

A

Thoracentesis to analyze it. Usually exudative with a lymphocytic predominance secondary to an enhanced immunologic response.

How well did you know this?
1
Not at all
2
3
4
5
Perfectly
15
Q

Acute jaundice, hepatomegaly, and elevated transaminases - DDx and patterns? Consideration for hospitalization?

A

Acute viral hepatitis (Hep A/B), alcoholic hepatitis (AST>ALT 3:2), toxic liver injury or drug-induced (acetominophen poisoning), autoimmune hepatitis (likely chronic onset), ischemic liver injury (in setting of trauma), herpesvirus etio (EBV, varicella, herpes simplex, CMV)

Hospitalize if unstable, poor f/u, acute risks, older, unable to tolerate PO

How well did you know this?
1
Not at all
2
3
4
5
Perfectly
16
Q

Young/middle-aged women who develop severe abdominal pain and have ascites on U/S - dx?

A

Budd-Chiari Syndrome due to hepatic vein obstruction (from thrombosis); thrombolytic therapy is a treatment option

How well did you know this?
1
Not at all
2
3
4
5
Perfectly
17
Q

Treatment considerations for acute hepatitis include?

A

Supportive - most can be followed with appropriate f/u and supportive measures if mildly symptomatic or asymptomatic; low risk of sig complications or fulminant hepatic failure w/ or w/o treatment

Antiviral therapy - for patients with immunosuppression, concurrent Hep C, severe hepatitis or fulminant hepatic failure

Lamivudine - for severe acute Hep B with impaired synthetic function (prolonged PT)

How well did you know this?
1
Not at all
2
3
4
5
Perfectly
18
Q

Treatment for Hep C?

A

Interferon alfa-2b and ribavirin

How well did you know this?
1
Not at all
2
3
4
5
Perfectly
19
Q

Risk of chronic hepatitis after acute Hep B infection? Best prognostic factor?

A

90% if perinatal transmission; 25-50% if within the first year of life

PT is the best prognostic factor for risk of developing chronic hepatitis. Elevated ALT > 6months also a sign.

How well did you know this?
1
Not at all
2
3
4
5
Perfectly
20
Q

Acute vs. Chronic Hep B labs?

A
Acute = HBsAg, HBeAg, IgM Anti-HBc, > ALT
Chronic = HBsAg with Anti-HBe, IgG anti-HBc
How well did you know this?
1
Not at all
2
3
4
5
Perfectly
21
Q

Child 6months - 5yrs old with sudden refusal to move arm after a pulling force - dx? Workup? Trmt?

A

Nursemaid’s elbow (radial head subluxation). No radiographs needed. Hyperpronation and Supination/Flexion = 2 primary reduction methods

How well did you know this?
1
Not at all
2
3
4
5
Perfectly
22
Q

Adequate INR level for someone on Warfarin?

A

INR 2-3

How well did you know this?
1
Not at all
2
3
4
5
Perfectly
23
Q

Important complication seen commonly in patients with hip fractures? How to prevent this?

A

DVT leading to fatal PE. LMWH or low-dose unfractionated heparin with continuation for 10-35 days depending on patient’s risk for thrombosis. It can be stopped 12hrs prior to surgery, low risk of bleeding

How well did you know this?
1
Not at all
2
3
4
5
Perfectly
24
Q

Pruritic rash on flexor surfaces typically starting on hands and spreading to abdomen, groin, skin folds, axilla, between fingers and toes - worse at night - excoriations with small, crusted red papules - dx? Workup? Trmt considerations?

A

Scabies - spread via contact; delayed type IV hypersensitivity to mites, ova, and feces.

Dx confirmed by skin scrapings.

Trmt Goals = mite eradication, decrease pruritis, and oral abx for any signs of secondary bacterial skin infection. If widespread, 5% topical permethrin over whole body; if localized, 1% terbinafine cream. Oral Ivermectin is also a treatment option for outbreaks in groups of people (nursing home). Oral antihistamines can help decrease dermatitis. Clothing/bedding cleaned and bagged for 3 days at minimum - mites cannot survive >3 days without human host.

How well did you know this?
1
Not at all
2
3
4
5
Perfectly
25
Q

Pruritic erythematous rash, circular, scaly lesions with central clearing - dx? Workup?

A

Tinea. Potassium hydroxide staining of skin sample can confirm dx by showing segmented hyphae and arthrospores.

How well did you know this?
1
Not at all
2
3
4
5
Perfectly
26
Q

Considerations for the workup of testicular trauma?

A

Minimal pain/swelling & Normal Exam - supportive care with f/u

Moderate pain/swelling - U/S with f/u, or furthered by findings

Severe pain/swelling and/or Abnormal PE - experienced U/S and expert surgical consultation

Get retrograde urethrography if blood seen at the meatus or if frank blood on urine dip

How well did you know this?
1
Not at all
2
3
4
5
Perfectly
27
Q

Screen for what in veterans with new onset insomnia, inadequate pain control, substance use, interpersonal conflicts? Trmt options?

A

PTSD. Trmt includes CBT and/or anti-depressants. Prazosin for nightmares

How well did you know this?
1
Not at all
2
3
4
5
Perfectly
28
Q

Elderly patients having difficulty with discriminating speech in noisy environments - dx?

A

Presbycusis - sensorineural hearing impairment

How well did you know this?
1
Not at all
2
3
4
5
Perfectly
29
Q

Difference in testing considerations b/w suspected active and latent TB?

A

Active (travel, cough, fever, night sweats, weight loss) - obtain chest imaging; abnormal or high suspicion continue with 3 sputum specimens for AFB and culture.

Latent - PPD or IGRA for patient’s with previous BCG vaccine. These both do not allow differentiation between active and latent

How well did you know this?
1
Not at all
2
3
4
5
Perfectly
30
Q

What can be used to tell if TB patient is infectious?

A

3 negative sputum smears. Patient could still have active TB though, just not infectious. Culture will dictate active TB but emperic therapy can be initiated if high suspicion.

How well did you know this?
1
Not at all
2
3
4
5
Perfectly
31
Q

Progressive dysphagia with solid, then liquids, in pt w/ h/o GERD - dx?

A

Esophageal Stricture - complication from healing process of ulcerative esophagitis in GERD pts.

vs. Achalasia = dysphagia of both solids AND liquids

How well did you know this?
1
Not at all
2
3
4
5
Perfectly
32
Q

Tricuspid vegetations, septic pulmonary emboli, systolic murmur increased with inspiration = dx?

A

Right sided endocarditis secondary to IV drug users. Peripheral manifestations are uncommon in these patients.

How well did you know this?
1
Not at all
2
3
4
5
Perfectly
33
Q

Bradykinesia, tremor, rigidity, unilateral onset of symptoms - dx? Trmt considerations?

A

Parkinsons - dopamine problem.

Initial therapy = dopamine agonists (Bromocriptine, Pramipexole) or Levadopa. However, levadopa can sometimes increase destruction of substantial nigra - begin in patients > 65yrs old with severe symptoms. Entacapone is a COMT inhibitor, helps by prolonging effect of levadopa.

How well did you know this?
1
Not at all
2
3
4
5
Perfectly
34
Q

Tremor in both hands/forearms that worsens with fine motor activity = dx? Trmt?

A

Essential tremor. Propanolol and primidone

How well did you know this?
1
Not at all
2
3
4
5
Perfectly
35
Q

Tamoxifen is used for?

A

Hormone responsive (estrogen +) BC and for the prevention of BC in woman at high risk. Associated w/ endometrial hyperplasia and cancer.

How well did you know this?
1
Not at all
2
3
4
5
Perfectly
36
Q

Side effects of the following supplements: Kava kava, Thiazide diuretics, Ginseng, Ginko?

A

Kava kava - hepatotoxicity, liver failure

Thiazides - hypokalemia, hyponatremia, hyperuricemia, elevated glucose and cholesterol

Ginko - bleeding 2/2 platelet and anticoagulant effects

Ginsing - HA, insomnia, GI symptoms, VB and hypoglycemia

How well did you know this?
1
Not at all
2
3
4
5
Perfectly
37
Q

Intracellular Gram-negative diplococci - treatment considerations?

A

Gonorrhea trmt = Ceftriaxone 250mg IM with concurrent Chlamydia trmt 2/2 coinfection rate. Oral doxycycline 100mg BID x7d or oral Azithromycin 1g single dose for Chalmydia trmt

How well did you know this?
1
Not at all
2
3
4
5
Perfectly
38
Q

Additional vaccinations for HIV + patients? Contraindicated vaccines?

A

Hep A and B unless previously documented immunity. Pneuomona vaccine PCV13 and PPSV23 8 weeks apart and every five years.

Contraindicated: lives vaccines (intranasal influenza - EXCEPTIONS are MMR and varicella zoster IF CD4 >200 and no hx of AIDS defining illness

How well did you know this?
1
Not at all
2
3
4
5
Perfectly
39
Q

Side effects of Isoniazid? MC and monitoring? Continuation of therapy?

A

Self-limited transaminitis, drug-induced lupus, nerve damage/muscle weakness.

MC = hepatotoxicity in 2 months; get baseline and monthly LFT’s

If asymptomatic but LFTs > 5x ULN OR Symptomatic with LFT’s >3x ULN- discontinue. Give B6 to prevent neuropathy.

How well did you know this?
1
Not at all
2
3
4
5
Perfectly
40
Q

Asymptomatic hyperuricemia associated with what TB drug?

A

Pyrazinamide - competes with uric acid excretion in kidneys

How well did you know this?
1
Not at all
2
3
4
5
Perfectly
41
Q

Side effect of ethambutol?

A

Ocular toxicity.

How well did you know this?
1
Not at all
2
3
4
5
Perfectly
42
Q

MC’ly responsible for drug-induced lupus (2 agents)?

A

Procainamide and hydralazine

How well did you know this?
1
Not at all
2
3
4
5
Perfectly
43
Q

AIDS patient with hemiparesis, speech/vision/gait disturbances with multiple demyelinating, non-enhancing lesions w/o mass effect, gradual onset - dx?

A

Progressive Multifocal Leukoencephalopathy - seen in immunocompromised patients via the JC virus - primarily involves cortical white matter. Dx with MRI.

VS. Toxo, CNS lymphoma, brain abscess with are ring-enhancing lesions w/ mass effect

VS. HIV encephalopathy w/ dementia as main sympt; similar MRI findings but bilateral and symmetric

How well did you know this?
1
Not at all
2
3
4
5
Perfectly
44
Q

MCC of fecal incontinence in elderly? Trmt?

A

Fecal impaction. Suppositories for complete emptying.

Would use rectal tube for acute pseudo obstruction resultng in dilated colon and abdominal distention

How well did you know this?
1
Not at all
2
3
4
5
Perfectly
45
Q

Trmt for grouped vesicles or bulla in specific unilateral dermatomal pattern with pain after bodily stress? Unresolved pain trmt months later?

A

Antiviral acyclovir or valacyclovir started within 72 hrs of onset of rash. Results from reactivation of lent infection in dorsal root ganglion.

Post-herpetic neuralgia trmt = amitriptyline (TCA), topical caspaicin, Gabapentin, and long acting oxycodone

How well did you know this?
1
Not at all
2
3
4
5
Perfectly
46
Q

Give varicella zoster vaccine s/p infection? Precautions?

A

If >60 yo, reduces the risk of zoster and post-herpetic neuralgia. If patient is immunocompetent, contact precautions with covering until lesions are completely crusted over 2/2 transmission from direct contact. If immunocompromised, hospitalized.

VS. primary varicella infxn (chickenpox) transmitted by airborne droplets

How well did you know this?
1
Not at all
2
3
4
5
Perfectly
47
Q

Vague symptoms like anorexia, decreased activity, irritability, abdominal pain, and insomnia in children should suspect was type of intox? Order what labs?

A

Lead! CBC, serum Fe, ferritin, reticulocyte count to assess for anemia and iron deficiency.

How well did you know this?
1
Not at all
2
3
4
5
Perfectly
48
Q

Possible drugs that could increase Lithium levels? Side effect of Lithium involving constipation and fatigue?

A

Thiazide diuretics, NSAIDs, ACE.

Lithium-induced hypothyroidism.

How well did you know this?
1
Not at all
2
3
4
5
Perfectly
49
Q

Two MC procedure for tattoo removal?

A

Dermabrasion and laser therapy

How well did you know this?
1
Not at all
2
3
4
5
Perfectly
50
Q

Continue anticoagulation for how long s/p initial DVT therapy?

A

3-6 months. Or lifelong if this is the second one.

D/c RF’s: obesity, smoking, OCP’s, HRT, Tamoxifen

How well did you know this?
1
Not at all
2
3
4
5
Perfectly
51
Q

What lab will prompt initiation of antibiotics in setting of Spontaneous Bacterial Peritonitis?

A

Neutrophil count >250 in the ascitic fluid

How well did you know this?
1
Not at all
2
3
4
5
Perfectly
52
Q

What formula used to calculate mortality rate in those with liver disease?

A

MELD Score = 3.8 (serum bilirubin) + 11.2 (INR) + 9.6(serum creatinine) + 6.4.

95% survival - score 40

How well did you know this?
1
Not at all
2
3
4
5
Perfectly
53
Q

Difference b/w breast milk jaundice and breastfeeding failure jaundice?

A

Breast Milk Jaundice - starts end of 1 week and peaks by 2 weeks; high B-glucuronidase in breast milk deconjugates intestinal bilirubin and thus increased level of unconjugated bili

Breastfeeding Failure Jaundice - bili secondary to decreased bili elimination; ppor latching, occurs first week, baby looks dehydrated

How well did you know this?
1
Not at all
2
3
4
5
Perfectly
54
Q

Recent asymmetric oligoarthritis that is inflammatory (elevated WBC’s) but sterile (culture negative) with a recent hx of infection - dx? Workup?

A

Reactive Arthritis - inflammatory (>2,000 WBC’s) synovial fluid extraarticular symptoms = urethritis, uveitis, circinate balantitis (painless shallow ulcers on glans penis persist for months). Retest for suspected infections even if asymptomatic or no evident signs.

Sources: GI (Salmonella, Shigella, Yersinia, Campylobacter, C Dif) and GU (Chlamydia)

Trmt - Abx and NSAIDS

How well did you know this?
1
Not at all
2
3
4
5
Perfectly
55
Q

Describe pathophys for neonatal breast hypertrophy and galactorrhea.

A

High estrogen crosses placenta in 3rd trimester; s/p delivery, estrogen drop off stimulates fetal prolactin production. Self-limited - 6 months

How well did you know this?
1
Not at all
2
3
4
5
Perfectly
56
Q

Episodes of well-circumscribed & raised erythematous plaques - associated intense pruritis - associated with angioedema in 40% - they worsen over minutes to hrs and dissipate within 24 hrs - dx? Trmt? Prog?

A

Chronic urticaria - from idiopathic stressor or stimulus.

Acute - H1/H2 blocker

Chronic - second generation H1 blocker (Loratadine, Cetirizine)&raquo_space; increase the dose&raquo_space; additional H1 blocker (hydroxyzine), leukotriene receptor antagonist (Montelukast), or H2 blocker (Ranitidine) o brief steroids

Prog - most have complete resolution 2-5 yrs

How well did you know this?
1
Not at all
2
3
4
5
Perfectly
57
Q

When to give Oseltamivir in pts with flu?

A

> 65yrs, pregnant, high risk medical comorbiditis - hospitalized patients or have involvement of lower respiratory tract - if they present within 48hrs of onset

How well did you know this?
1
Not at all
2
3
4
5
Perfectly
58
Q

Recent abdominal sx (2-3wks), swinging fevers, leukocytosis, right shoulder tip pain, cough - dx? Workup?

A

Subphrenic Abscess. Abdominal U/S

How well did you know this?
1
Not at all
2
3
4
5
Perfectly
59
Q

Types of hypersensitivity?

A

Type I - IgE mediated, release of vasoactive mediators from mast cells and basophils (anaphylaxis)

Type II - IgG mediated, antibody attack with complement (autoimmune hemolytic anemia)

Type III - Antibody-complex deposition causing damage (serum sickness, PAN, glomerulonephritis in Hep B)

Type IV - Cell mediated; memory cell activation of macrophages - delayed hypersensitivity (contact dermatitis)

How well did you know this?
1
Not at all
2
3
4
5
Perfectly
60
Q

Likelihood ratio of having and not having the disease given a positive/negative result respectively? Formula?

A

Likelihood ration:
(+) = sensitivity / (1 - specificity)

(-) = (1 - sensitivity) / specificity

How well did you know this?
1
Not at all
2
3
4
5
Perfectly
61
Q

Elevated TSH and normal T4 w/o symptoms, dx? Trmt necessary? Workup?

A

Subclinical hypothyroidism. Treat if: +antithyroid ab’s, abnormal lipid profile, symptomatic, ovulatory/menstrual dysfxn

Workup: get anti-thyroid ab’s; only get U/S if abnormal PE finding

How well did you know this?
1
Not at all
2
3
4
5
Perfectly
62
Q

How to calculate RR when only one relation is known?

A

If you know the RR of A compared to B = x, then the RR of B compared to A = 1/x

How well did you know this?
1
Not at all
2
3
4
5
Perfectly
63
Q

Saw palmetto = alternative med trmt for? St. John’s wort? OA? Garlic?

A

Saw palmetto - BPH
St. John’s Wort - anxiety/depression
Garlic - hypercholesterolemia
Glucosamine and Chondoitin = OA

How well did you know this?
1
Not at all
2
3
4
5
Perfectly
64
Q

Popular alternative meds with increased bleeding risk?

A

Ginko biloba, saw palmetto, ginseng, black cohosh, and garlic

How well did you know this?
1
Not at all
2
3
4
5
Perfectly
65
Q

St John’s Wort side effects?

A

GI distress, dizziness, fatigue, photosensitivity, and dry mouth

How well did you know this?
1
Not at all
2
3
4
5
Perfectly
66
Q

2 MCC of diaphragmatic paralysis in the newborn = ?

A

Birth injury and cardiothoracic injury causing damage to phrenic nerve

How well did you know this?
1
Not at all
2
3
4
5
Perfectly
67
Q

Strongest known risk factor for male BC?

A

Klinefelter Syndrome = 47 XXY, resultant hypogonadism, low T, and gynecomastia

How well did you know this?
1
Not at all
2
3
4
5
Perfectly
68
Q

CVS sampling best done at what GA? why?

A

> 11 wks GA 2/2 lowest risk for limb reduction anomaly

How well did you know this?
1
Not at all
2
3
4
5
Perfectly
69
Q

Neurologic symptoms that vary in time and space with intermittent recovery - common symptom = optic neuritis (monocular vision loss and eye pain w/ movmt) - dx? Best test? If atypical, helpful test?

A

Multiple Sclerosis. MRI shows ovoid periventricular white matter lesions. If unsure, LP will show elevated oligoclonal bands and elevated IgG index.

MS patients can also develop transverse myelitis = UMN signs and sensory deficits below the level of the lesion

How well did you know this?
1
Not at all
2
3
4
5
Perfectly
70
Q

Trmt considerations for MS?

A

Acute exacerbation = IV/Oral Corticosteroids

If optic neuritis involved, IV route b/c increased risk of recurrence with PO steroids

Refractory to glucocorticoids = plasmapharesis

Maintenance = disease-modifying = beta-interferon and glatiramer acetate are indicated for chronic maintenance in relapsing-remitting MS

Treated similarly in pregnancy - higher rate of assisted deliveries

How well did you know this?
1
Not at all
2
3
4
5
Perfectly
71
Q

Management of MS comorbidities? Depression, fatigue, muscle spasticity, neuropathic pain, urge incontinence…

A

Depression - SSRI’s
Spasticity - Baclofen, massage
Fatigue - Amantadine
Neuropathic pain - Gabapentin or duloxetine
Urge Incontinence - Oxybutynin, timed voiding

How well did you know this?
1
Not at all
2
3
4
5
Perfectly
72
Q

Diagnostic considerations for Allergic Bronchopulmonary Aspergillosis? Presentation? Trmt and monitoring?

A

Hx of Asthma, + skin test reactivity to Aspergillus, + serum Ab’s, serum IgE concentration > 1000, eosinophilia, bilateral upper lobe infiltrates, central bronchiectasis.

Intense IgE and IgG response in asthmatic patients with ABPA causes fever, malaise, cough w/ brown expectorant, wheezing, and signs of bronchial obstruction

Treat with oral steroids (NOT inhaled) - follow serum IgE concentration and clinical symptoms for improvement

How well did you know this?
1
Not at all
2
3
4
5
Perfectly
73
Q

3 considerations to help distinguish cause of hypoglycemia?

A

Serum insulin, C-peptide, and hypoglycemic drug assay:

  • insulinoma will have negative drug assay; U/S or CT to investigate (or 72hr fasting challenge)
  • exogenous insulin will have negative c-peptide and assay
  • exogenous hypoglycemia agent will have + assay
How well did you know this?
1
Not at all
2
3
4
5
Perfectly
74
Q

Considerations for giving tetanus rx’s w/ injuries?

A

If the patient’s hx is good (has >3 immunizations):

  • minor/clean cut: only give tetanus toxoid IF last shot was >10 yrs ago
  • major/dirty cut: only give tetanus toxoid IF last shot was > 5 yrs ago

If the pt’s hx is bad (

How well did you know this?
1
Not at all
2
3
4
5
Perfectly
75
Q

Proximal symmetric muscle weakness in upper/lower extremities, Gottron’s papules, heliotrope rash, ILD, dysphagia, myocarditis = dx? Work up? Trmt?

A

Dermatomyositis. Elevated CPK, Anti-RNP, Anti-Jo-1, Anti-Mi2 (EMG and muscle biopsy if uncertain)

Trmt = high dose glucocorticoids + glucocorticoid-sparing agent

** Screen for malignancy ** Adenocarcinoma of cervix, ovaries, pancreas, bladder, lung, stomach

How well did you know this?
1
Not at all
2
3
4
5
Perfectly
76
Q

Mostly seen in women, 30-65 yo, with milady elevated LFT’s with elevated Alk Phos, itching - dx? Tests? Trmt? Screen for what complication?

A

Primary Biliary Cirrhosis - autoimmune problem of small to mid-sized bile ducts - causing progressive fibrosis, ESLD

Antimitochondrial Ab’s (AMA) = high sensitivity and specificity; diagnostic confirmation requires liver bx

Can see xanthelasma from elevated LDL and hypercholesterolemia

Trmt = ursodeoxycholic acid / transplant

Complication = Osteoporosis via bone densitometry 2/2 malabsorption of fat soluble vitamins A, D, E, K

How well did you know this?
1
Not at all
2
3
4
5
Perfectly
77
Q

Anti-smooth muscle Ab’s association = ?

A

Type 1 Autoimmune Hepatitis

How well did you know this?
1
Not at all
2
3
4
5
Perfectly
78
Q

Liver disease, skin hyper pigmentation, DM, arthropathy, cardiac involvement, and hypogonadism - dx? Abnormal lab profile?

A

Hemochromotosis - abnormal deposition of Fe. Elevated LFT’s > Alk Phos

How well did you know this?
1
Not at all
2
3
4
5
Perfectly
79
Q

Treatment for patients that have white/blue discolored fingers in the cold? Additional workup?

A

Dihydropyridine Ca-channel blockers (nifedipine/amlodipine) and diltiazem; nitroglycerin in refractory pts

Workup with ANA, RF, CBC, blood chem, UA and complement levels in pts with systemic manifestations

How well did you know this?
1
Not at all
2
3
4
5
Perfectly
80
Q

How can an infant get botulism? Trmt considerations?

A

Infant vs. Foodborne Botulism via ingestion of Clostridium Botulinum spores from environmental dust vs. performed c. botulinum toxin with canned foods.

Human-derived botulism Immune Globulin vs. Equine-derived botulism antitoxin

Toxin inhibits presynaptic release of cholinergic transmission. It can predominate in infant GI flora b/c still developing flora.

Higher in CA, PA, UT - disturbed soil (farming/construction)

Most infants require hospitalization for 1-3 months with complete recovery

How well did you know this?
1
Not at all
2
3
4
5
Perfectly
81
Q

HIV rxs in pregnancy?

A

HAART - 2NRTI’s (Zidovudine and Lamivudine) + 1 protease inhibitor or NNRTI; Efavirenz is preferred after 8th wk gestation b/c risk of NT, cleft lip, anopthalmia. However, DONT CHANGE AN EFFECTIVE REGIMEN IF THE PATIENT STARTED IT ALREADY - risk of transmission from changing a regimen outweighs risk of teratogenicity. Reduces rate of transmission from 25% to 2%

How well did you know this?
1
Not at all
2
3
4
5
Perfectly
82
Q

Delivery considerations with HIV+ its?

A

C/S if VL> 1,000. UD viral load and complaint with HARRT can have vaginal delivery

How well did you know this?
1
Not at all
2
3
4
5
Perfectly
83
Q

Reasons to not BF?

A

HIV infection (unless in developing country), active TB (can start after 2 wks s/p trmt initiation), herpetic breast lesions, Varicella infxn

How well did you know this?
1
Not at all
2
3
4
5
Perfectly
84
Q

Causes of acquired long QT Syndrome?

A

Bradyarrhythmias = MCC of Torsades

Meds: diuretics (electrolyte imbalance), antipsychotics, tricyclic antidepressants, antiarrhythmics (amiodarone, sotalol, flecainide), anti-infective drugs (macrolides, fluoroquinolones

Other: hypothermia, hit, MI

How well did you know this?
1
Not at all
2
3
4
5
Perfectly
85
Q

% correlations for std deviations 1, 2, and 3?

A

68%, 95%, 99.7%

How well did you know this?
1
Not at all
2
3
4
5
Perfectly
86
Q

Risk factors for invasive aspergillosis? Presents with?

A

Allogenic transplantation, older age, acute graft-vs-host disease and corticosteroid therapy. Fever, sinus symptoms, and pulmonary complaints

How well did you know this?
1
Not at all
2
3
4
5
Perfectly
87
Q

Formula for NNT?

A

1/ARR = NNT

ARR = percentage of relapse/disease in treated group subtracted from placebo group

How well did you know this?
1
Not at all
2
3
4
5
Perfectly
88
Q

Common side effects of:

1) CCB’s
2) ACEi’s
3) HCTZ

A

1) Lower extremity edema 2/2 precapillary vasodilatation (25% with DHP’s like Amlodipine/Nifedipine in 6mo’s of therapy); headache, flushing, dizziness
2) Angioedema (0.1-0.7%)
3) Hyponatremia, hypokalemia, renal failure, hyperuricemia (acute gout),. and elevated glucose and lipid levels

How well did you know this?
1
Not at all
2
3
4
5
Perfectly
89
Q

What is Type II Polyglandular Autoimmune Failure?

A

Addison’s disease w/ Type 1 DM

How well did you know this?
1
Not at all
2
3
4
5
Perfectly
90
Q

Considerations for breast mass work up?

A
  • Spontaneous, unilateral, and bloody = concerning and warrants work up with mammo (MCC of pathologic nipple discharge = papillary tumor)
  • Bilateral milky white discharge should first be assessed with prolactin levels 2/2 hyperprolactinemeia from prolactinoma, medications (antipsychotics), hypothyroidism, and pregnancy
How well did you know this?
1
Not at all
2
3
4
5
Perfectly
91
Q

RF for developing reactive arthritis?

A

HLA-B27 positive with gram-negative rods

How well did you know this?
1
Not at all
2
3
4
5
Perfectly
92
Q

Athletic women with secondary ammenorrhea have low levels of what three hormones?

A

GnRH, LH, and estrogen

How well did you know this?
1
Not at all
2
3
4
5
Perfectly
93
Q

Common causes of recurrent sinusitis?

A

Smoke exposure (via damage to cilia), inadequately treated acute sinusitis. structural abnormalities of palate or septum, and allergic rhinitis

How well did you know this?
1
Not at all
2
3
4
5
Perfectly
94
Q

Monitor what value to reassess asthmatics in ED? Considerations?

A

PEF:

  • > 70% and no distress = good to d/c
  • 40-69% = admit to hospital ward
  • 42 and severe signs = admit to ICU
How well did you know this?
1
Not at all
2
3
4
5
Perfectly
95
Q

Treatment of syphylis by stage?

A
  • Primary, Secondary, Early Latent (12 months), unknown, gummatous/CV sylph = Benzathine PCN 2.4M IM weekly x 3 wks
  • Neurosyph = Aqueos PCN 3-4M IV q4hrs for 14 days
  • Congenital Syph = Aqueos PCN 50,000units/kg IV q 8-12hrs for 10 days
How well did you know this?
1
Not at all
2
3
4
5
Perfectly
96
Q

Rxn developing within 24hrs after initiation of treatment for spirochetal infxn = dx? Prevention?

A

Jarisch-Herxheimer Rxn - no prevention. constituted by fever + malaise/chills/HA/myalgias

How well did you know this?
1
Not at all
2
3
4
5
Perfectly
97
Q

RF’s for Lithium toxicity? Levels? Presents?

A

Low GFR (elderly, renal pts), volume depletion, drug interactions (thiazide diuretics, ACEi’s, NSAIDs)

Sympt = confusion, ataxia, neuromuscular excitability (tremor), N/V/D

Trmt = Lithium levels q2hrs, IV hydration, hemodialysis

How well did you know this?
1
Not at all
2
3
4
5
Perfectly
98
Q

Pathophys for:

  • Toxic Shock Syndrome
  • Bacterial Endocarditis
  • Septic Shock
A
  • TSS = (MCC Staph Aureus) exotoxin (TSS toxin-1) causes widespread activation of T cells, acting like super antigens - leading to massive cytokine production
  • Endocarditis = bacterial overgrowth and dissemination
  • Septic Shock = massive bacterial lysis and subsequent circulating endotoxin
How well did you know this?
1
Not at all
2
3
4
5
Perfectly
99
Q

Treatment of Toxis Shock Syndrome =

A

Fluid replacement (up to 20L/day), Clindamycin theoretically prevents toxin synthesis, and Antistaph antimicrobial therapy (vancomycin, oxacillin or nafcillin)

How well did you know this?
1
Not at all
2
3
4
5
Perfectly
100
Q

Frequency of UTI’s to start prophylaxis? Meds?

A

2 in six months or 3 in 1yr

Bactrim, fluoroquinolones, and Nitrofurantoin

How well did you know this?
1
Not at all
2
3
4
5
Perfectly
101
Q

What thionamide rx’s to use in pregnancy and at what GA? Why?

A

Propylthiouracil (PTU) - risk of liver disease - use in 1st trimester

Methimazole (MMI) - teratogenicity - use in 2nd/3rd trimester

How well did you know this?
1
Not at all
2
3
4
5
Perfectly
102
Q

Whipple’s triad confirming true hypoglycemia?

A

1) low BG 2) symptomatic 3) relief of symptoms with glucose

How well did you know this?
1
Not at all
2
3
4
5
Perfectly
103
Q

Advice for hypoglycemia during exercise for diabetics with insulin reqmt?

A

Exercise increases insulin uptake by exercising muscles so avoid injections into exercising limbs, can try snack before exercising, and decrease prior dosage. Check BG before and after exercising

How well did you know this?
1
Not at all
2
3
4
5
Perfectly
104
Q

Different presentation of Giardia based on acuity? Trmt and prevention?

A

Asymptomatic - shed cysts for 6 months

Acute - diarrhea, steatorrhea, flatulence, N, F, weight loss

Trmt - not for asymptomatic; if symptomatic, use Metronidazole, nitazoxanide, or tinidazole.

Prevent by contact isolation and hand hygiene

How well did you know this?
1
Not at all
2
3
4
5
Perfectly
105
Q

What’s a positive apnea test?

A

Can confirm brain death: absent respiratory response off the vent for 8-10 minutes with a PaCO2 >60mmHg and a final aerial pH

How well did you know this?
1
Not at all
2
3
4
5
Perfectly
106
Q

Symptoms of B12 def? Complication with hematologic phenomena?

A

Dementia, subacute combined degeneration, dorsal spinal column probs (loss of vibratory sense, + Romberg test) and lateral corticospinal tract abnormalities (spastic paresis, hyperreflexia)

Secondary to poor DNA synth in RBC’s, they mature improperly and are megaloblastic, eventually lysing in the bone marrow and causing indirect hyperbili (can see decreased haptoglobin, elevated LDH)

How well did you know this?
1
Not at all
2
3
4
5
Perfectly
107
Q

Difference b/w nightmare d/o and night terrors?

A

Night terrors = non-REM poor/little recollection

Nightmare D/O = REM stage d/o and child has vivid dream recall

How well did you know this?
1
Not at all
2
3
4
5
Perfectly
108
Q

Symptoms of carbon monoxide poisoning and lab?

A

Multiple people having fatigue, malaise, throbbing HA’s, nausea. Get a carboxyhemoglobin

How well did you know this?
1
Not at all
2
3
4
5
Perfectly
109
Q

Knee pain, decreased ROM, skin changes, and vasomotor changes (skin color changes) after a recent injury = dx? Trmt? Pathogenies?

A

Complex Regional Pain Syndrome.

Patho = nerve injury causing increased sensitivity to sympathetic nerves, abnormal response to and sensation of pain, and increased neuropeptide release causing allodynia

How well did you know this?
1
Not at all
2
3
4
5
Perfectly
110
Q

Presentation of HSV encephalitis?

A

Hemorrhagic infarction of the temporal love with elevated RBC on CSF analysis

How well did you know this?
1
Not at all
2
3
4
5
Perfectly
111
Q

Patients are at risk for what who have history of chronic lymphocytic infiltration of the thyroid? Presentation? What PE test?

A

Hashimotos is a RF for Thyroid lymphoma. Rapid and acute swelling of thyroid area. Pemberton’s test = raising hands over head and + if patient becomes plethoric, indicating thyroid is the cause of obstructive symptoms.

How well did you know this?
1
Not at all
2
3
4
5
Perfectly
112
Q

A1c goal range for DM2?

A

6-7%

How well did you know this?
1
Not at all
2
3
4
5
Perfectly
113
Q

Contraindication to HPV vaccine?

A

Hypersensitivty to yeast

How well did you know this?
1
Not at all
2
3
4
5
Perfectly
114
Q

RF’s for intussusception? Presentation? Dx and trmt?

A

Recent viral illness or rotavirus vaccination, polyps, tumors, HSP, celiac disease, meckels. Occurs 6mo - 3yr. Episodic crampy abdominal pain, sausage mass, currant jelly stools. Target sign on U/S = clue. Don’t need imaging if clear dx from presentation - use air or water soluble enema.

How well did you know this?
1
Not at all
2
3
4
5
Perfectly
115
Q

Distinguishing characteristics and labs of CAH types? Trmt?

A

21-hydroxylase = ambiguous genitalia in girls, hypotension, hypoglycemia 2/2 increased testosterone (shunted products), decreased cortisol and aldosterone causing hypoglycemia, hyperkalemia, hyponatremia. See elevated 17-hydroxyprogesterone. = MC type of CAH. Give glucocorticoids and mineralocorticoids

11B-hydroxylase = HTN

17a-hydroxylase = HTN

How well did you know this?
1
Not at all
2
3
4
5
Perfectly
116
Q

Findings in gout arthro? vs. pseudogout?

A

Monosodium urate crystals = negatively birefringent, needle shaped crystals under polarizing light (yellow)

Pseudogout = positively birefringent rhomboid shaped crystals (blue) CPPD crystals (calcium pyrophosphate dihydrate); more likely to present in RA or OA - MC’ly seen in knees

How well did you know this?
1
Not at all
2
3
4
5
Perfectly
117
Q

Trmt considerations in gout?

A

NSAIDS (Indomethicin) - contra = AKI/CKD, CHF, PUD, Anticoagulation

Colchicine - contra = severe or liver disease

Intraarticular steroids - contra = >2 joints involved

How well did you know this?
1
Not at all
2
3
4
5
Perfectly
118
Q

Types of common causes of male urethritis? Trmt considerations?

A

Gonorrhea - purulent, gram negative rods
Chlamydia - PCR, give azithro
Trich - metronidazole
Mycoplasma Genitalium - covered by azithro
Ureaplasma - covered by azithro

How well did you know this?
1
Not at all
2
3
4
5
Perfectly
119
Q

Differential for moving paralysis?

A

Tick paralysis - normal CSF, occurs hours to days (remove tick)
GBS - days to weeks, elevated CSF protein w/ normal WBC (IV immunoglobulin or plasmapharesis)
Myasthenia gravis - not ascending paralysis, end of day onset (acetyl cholinesterase inhibitors)
Botulism - descending paralysis

How well did you know this?
1
Not at all
2
3
4
5
Perfectly
120
Q

Side effects of amiodarone?

A

Thyroid dysfunction, corneal deposits, skin discolorations, pulmonary fibrosis, and liver toxicity

How well did you know this?
1
Not at all
2
3
4
5
Perfectly
121
Q

When do you give prophylactic acyclovir to pregnant patients?

A

At 36wks GA if they have hx of HSV; c/s if active at term.

If no hx but exposed, type specific antibody testing for HSV1/2 is appropriate to determine infections; if negative nothing is needed, but if + can begin prophylactic trmt at 36 wks

How well did you know this?
1
Not at all
2
3
4
5
Perfectly
122
Q

MCC of malignant otitis media? Trmt options?

A

Pseudomonas

Fluoroquinolones = cipro
PCN’s - Piperacillin, Ticarcillin
3rd Gen Cephalosporins = Ceftazidime

Should start trmt IV then switch to PO

How well did you know this?
1
Not at all
2
3
4
5
Perfectly
123
Q

What is Ramsay Hunt Syndrome?

A

Reactivated zoster virus causing ear pain, vesicles in external auditory canal and ipsilateral facial paralysis

How well did you know this?
1
Not at all
2
3
4
5
Perfectly
124
Q

Characteristics of: NF1, NF2, Tuberous Sclerosis, Sturge Weber Syndrome, Osler Rendu Weber?

A

NF1 - unilateral acoustic neuromas, cutaneous neurofibromas, axillary freckling, hyperpigmented cafe-au-lait spots
NF2 - hypo pigmented spots, hx of bilateral deafness from bilateral acoustic neuromas
SW - facial port wine and leptomeningeal angiomatosis
TS - congenital hypopigmented maculae (ash leaf), glial proliferation, and several organ hamartomas/cysts
ORW - multiple telangiectasias and vascular lesions of CNS

How well did you know this?
1
Not at all
2
3
4
5
Perfectly
125
Q

Common type of PTHrP releasing malignancy?

A

Squamous Cell Carcinoma

How well did you know this?
1
Not at all
2
3
4
5
Perfectly
126
Q

Describe Hand-Foot-Mouth Disease. Cause?

A

Exanthem can be on palms, soles, genitalia, and/or buttocks with herpangina (vesicles on posterior oropharynx). Caused by Group A Coxsackievirus

How well did you know this?
1
Not at all
2
3
4
5
Perfectly
127
Q

Intoxication causing: confusion, lethargy, bradycardia, skin flushing, miosis, wheezing, garlic like odor?

A

Organophosphate poisoning. Test with RBC cholinesterase

How well did you know this?
1
Not at all
2
3
4
5
Perfectly
128
Q

Treatment of ACS?

A

MONA HABS Clopidogrel - Morphine, Oxygen, Nitrates, Aspirin, Heparin, B-blockers, Statin, Clopidogrel

  • 2 antiplatelet therapies = aspirin and clopidogrel
How well did you know this?
1
Not at all
2
3
4
5
Perfectly
129
Q

Risk factors for elder abuse?

A

Female gender, mental/physical illness, and old age.

How well did you know this?
1
Not at all
2
3
4
5
Perfectly
130
Q

Factors protected by confidentiality in minors:

A

pregnancy, contraception, substance use, STD’s, psych illness - UNLESS they become a danger to themselves

How well did you know this?
1
Not at all
2
3
4
5
Perfectly
131
Q

Molluscum contagiousum is caused by what virus? spread? Suspect any additional infections?

A

Poxvirus - skin to skin contact. Test for HIV if these are widespread and/or involve the face

How well did you know this?
1
Not at all
2
3
4
5
Perfectly
132
Q

How to treat chlamydia in pregnancy?

A

Erythromycin base 500mg PO QD x7days
Amoxicillin 500mg PO TID x7days

Nonpregnant: azithromycin 1g PO, doxycycline 100mg BID x7days

How well did you know this?
1
Not at all
2
3
4
5
Perfectly
133
Q

Non-SSRI antidepressant options for people with sexual dysfunction on SSRI’s?

A

Bupropion or Mirtazapine; can consider adding sildenafil (phosphodiesterase-5 inhibitor)

How well did you know this?
1
Not at all
2
3
4
5
Perfectly
134
Q

Dx for decreased sensation over the anterolateral thigh without any muscle weakness or DTR abnormalities - hx of chronic flexion at waist? Nerve involved?

A

Meralgia paresthetica - entrapment of lateral femoral cutaneous nerve (purely sensory, direct branch from lumbar plexus runs under inguinal ligament)

How well did you know this?
1
Not at all
2
3
4
5
Perfectly
135
Q

T-scores for assessing bone mineral density? via DEXA. When to add rx’s?

A

above - 1 = normal
-1 to -2.5 = osteopenia
below -2.5 = osteoporosis

Pharm therapy with T less than -2.5 or those with fragility fractures (hip/vertebral fractures from low-trauma) = bisphosphonates generally 1st line (Alendronate decreases bone resorption) + 1200mg/day elemental calcium and 800IU/day Vitamin D

How well did you know this?
1
Not at all
2
3
4
5
Perfectly
136
Q

Episodic HA, young age htn, refractory htn, adrenal mass, sweating, tachycardia = screen for? Via? If elevated next steps?

A

Pheochromocytoma with plasma free metanephrine levels or 24-hr urine collection for catecholamine and metanephrine

If elevated: CT/MRI of abdomen and surgical veal, genetic testing, alpha (maybe beta) blockade w/ phenoxybenzamine prior to surgery, MIBG (functional scintigraphy w/ norepi-like substance) scan if >5cm. Surgery only after 10-14 days of BP control, IV fluids.

If elevated and negative imaging, get MIBG to assess for undetected tumors

How well did you know this?
1
Not at all
2
3
4
5
Perfectly
137
Q

Complications with pheochromocytoma surgery and how to handle them?

A

1) HTN Crisis - IV nitroprusside, phentolamine, or nicardipine
2) Hypotension - normal saline bolus, pressers if unresponsive
3) Hypoglycemia - IV dextrose
4) Cardiac tachyarrhythmias - IV lidocaine or esmolol

How well did you know this?
1
Not at all
2
3
4
5
Perfectly
138
Q

Cardiac resynchronization w/ biventricular pacing device is recommended in pts w/: ???

A

Sinus rhythm and: severe LVSD w/ EF 150msec)

How well did you know this?
1
Not at all
2
3
4
5
Perfectly
139
Q

Next step in pulseless electrical activity? What rhythms are shockable? Cardiovert?

A

CPR. Defibrillate someone with Vfib or pulseless Vtach. Can cardiovert with unstable tachycardia (has pulse still)

How well did you know this?
1
Not at all
2
3
4
5
Perfectly
140
Q

Indications for IVC filter? Acute/longterm risks?

A

Indicated in those who have contraindications to anticoagulation (recent surgery, hemorrhagic stroke, bleeding prob, active bleeding). Acute probs = insertion site thrombosis, hematoma, AV fistula. Chronic probs = recurrent DVTs (stops them from traveling but not their recurrence)

How well did you know this?
1
Not at all
2
3
4
5
Perfectly
141
Q

Hyperglycemia, metabolic acidosis, + ketones = dx? Important formula? Therapy algorithm.

A

DKA - Anion Gap = (Na - [Cl + HCO3-]), normal = 7-13.

Initially, IV fluid support with continuous IV insulin (be wary of hypokalemia); add to K to fluids if K 135.

Insulin: switch to SQ when patient can eat, glucose 15. Overlap SQ and IV by 1-2hrs

Bicarb: if pH

How well did you know this?
1
Not at all
2
3
4
5
Perfectly
142
Q

What to start in asthmatic exacerbation?

A

Immediate O2 > inhaled B2 agonist > systemic IV steroids

Life threatening exacerbations, IV Magnesium

How well did you know this?
1
Not at all
2
3
4
5
Perfectly
143
Q

MC type of bias in case-control studies?

A

Recall bias - those with disease more likely to report exposure.

How well did you know this?
1
Not at all
2
3
4
5
Perfectly
144
Q

Unilateral HA’s occuring 1-3x daily, lasts 30min - 3hrs, severe retroorbital pain, lacrimation, conjunctival injection, rhinorrhea, sweating, and pallor occurring for 4-8 wks at a time = dx? Prevention? Abortifactant?

A

Cluster HA. Verapamil can be used for prevention with duration of HA’s longer than two months (Lithium if refractory). Can use prednisone if expected use is

How well did you know this?
1
Not at all
2
3
4
5
Perfectly
145
Q

When to use injection sclerotherapy for varicose veins?

A

Failed 3-6 months of conservative treatment. Try conservative measures first: leg elevation, wt loss, compression stockings (don’t use if they have underlying arterial insufficiency)

How well did you know this?
1
Not at all
2
3
4
5
Perfectly
146
Q

Hemodynamic measurements of heart?

A
R Atrial P = 4
PCWP (L atrial surrogate) = 9
Cardiac Index (pump fxn) = 2.8-4.2
SVR = 1150

Septic shock has low SVR and increased CO = differentiates this from cardiogenic and hypovolemic causes. While increased R atrial and PCWP are increased in cardiogenic secondary to back up.

How well did you know this?
1
Not at all
2
3
4
5
Perfectly
147
Q

3 criteria for Lewy Body Disease? Trmt?

A

Cognitive FLUCTUATIONS, visual hallucinations, and parkinsonism. Mix of Alzheimers and Parkinsons.

Trmt = cholinesterase inhibitors (AD) and levadopa and dopamine agonists (PD)

How well did you know this?
1
Not at all
2
3
4
5
Perfectly
148
Q

Dx: disinhibition, personality changes, extremem agitation, and urinary incontinence?

A

Frontotemporal dementia - occurs in 50’s.

How well did you know this?
1
Not at all
2
3
4
5
Perfectly
149
Q

Urge to move legs and is exacerbated with inactivity, relieved with movement = dx? Other causes? Trmt?

A

Restless leg syndrome. Secondary causes = iron deficiency anemia, uremia, DM, MS, pregnancy, antidepressants.

Trmt - start with iron if ferritin low

How well did you know this?
1
Not at all
2
3
4
5
Perfectly
150
Q

Diagnostic criteria for acute pancreatitis? Workup for etiology? Complication if the patient deteriorates days later with supportive care?

A

Classical symptoms of abdominal pain radiating to the back with elevated amylase and lipase (more specific). CT not absolutely necessary.

Alcohol is leading cause; can order U/S if suspecting gallstones, LFT’s, Calcium, or lipid panel for hypertriglyceridemia

Pancreatic necrosis can cause SIRS.

How well did you know this?
1
Not at all
2
3
4
5
Perfectly
151
Q

Treat this first in AF?

A

Rate control with beta blockers.

How well did you know this?
1
Not at all
2
3
4
5
Perfectly
152
Q

What is the Standardized Incidence Ratio?

A

Used to see if the incidence is higher in a particular group in relation to larger population = Observed Cases / Expected Cases

How well did you know this?
1
Not at all
2
3
4
5
Perfectly
153
Q

Treatment for Hidradenitis Suppurativa?

A

RFs = hx of HS, obesity, smoking, skin stress

Topical Clindamycin or oral antibiotics for flare up in those with initial; solitary lesion

Abscess formation and purulent sinus tracts oral tetracyclines are preferred (Doxy)

Diffuse tracts and severe disease = infliximab (tnf alpha inhibitors), surgical excision

How well did you know this?
1
Not at all
2
3
4
5
Perfectly
154
Q

HIV testing timeline after exposure? How often to check CD4 and VL in confirmed cases?

A

Using ELISA (HIV RNA PCR) or test for p24 antigen at 6, 12, and 24wks. Check every 3-4months

How well did you know this?
1
Not at all
2
3
4
5
Perfectly
155
Q

Considerations for who to take to the cath lab?

A

Take STEMI patients immediately. Take NSTEMI in 24-48 hrs (differentiated from unstable angina with elevated troponins). Cardiac enzymes might not rise for 6 hrs!!!! Need to monitor! Stress test those after r/o acute MI

How well did you know this?
1
Not at all
2
3
4
5
Perfectly
156
Q

Periarticular erosions, elevated RF levels, symmetric swelling and mild hyperemia of the proximal small hand joints, wrists and knees = dx? Treatment considerations?

A

RA. Start with methotrexate; if refractory to this + steroids for 6 months, can use infliximab or etanercept (get TB testing first!!!!); if refractory can try hydroxychloroquine, methotrexate, and sulfasalzine OR switching to cyclosporine

How well did you know this?
1
Not at all
2
3
4
5
Perfectly
157
Q

MCC of hemoptysis? Trmt?

A

Acute bronchitis; antibiotics

How well did you know this?
1
Not at all
2
3
4
5
Perfectly
158
Q

Window of time for tPA? Considerations prior to therapy?

A

Given less than 3-4.5hrs after the onset of symptoms. Assess for: active internal bleeding, platelets 185/110

How well did you know this?
1
Not at all
2
3
4
5
Perfectly
159
Q

Recurrent lapses into sleep, napping multiple times in day at least 3x/wk for 3 months with cataplexy by trigger, REM sleep latency

A

Narcolepsy. Get a polysomnography. Modafenil.

Can also use SNRI (Venlafaxine) or SSRI, or TCA if refractory treatment with cataplexy specifically.

How well did you know this?
1
Not at all
2
3
4
5
Perfectly
160
Q

Protective factors in SI?

A

Social support/family connectedness, Pregnancy, Parenthood, Religion

How well did you know this?
1
Not at all
2
3
4
5
Perfectly
161
Q

How to decide when to treat strep?

A

CENTOR Criteria: tonsillar exudates, anterior cervical lymphadenopathy, fever, absence of cough.

If 1 or less = treat symptoms
If 2 = get rapid strep test
If 3 or more = 50% PPV for strep

How well did you know this?
1
Not at all
2
3
4
5
Perfectly
162
Q

Elevated glucose levels without ketones in a DM2 patient = dx? Start by giving?

A

Hyperosmolar hyperglycemic state - FLUIDS (8-10L behind)

How well did you know this?
1
Not at all
2
3
4
5
Perfectly
163
Q

Pathophys for pernicious anemia? What to monitor after starting B12 in patients?

A

Atrophic gastritis secondary to autoimmune destruction of parietal cells, leading to achlorhydria, and decreased production of intrinsic factor. This causes less binding of B12 which then cannot be taken to terminal ileum for absorption.

Monitor K because sudden uptake of K for newly formed RBC’s can be life threatening.

How well did you know this?
1
Not at all
2
3
4
5
Perfectly
164
Q

Acromegaly long term risks?

A

Cardiac disease!!! DM, respiratory probs, increased colon cancer risk.

How well did you know this?
1
Not at all
2
3
4
5
Perfectly
165
Q

Screen for what in patients with short stature, hypogonadism, short 4th metacarpal bones, multiple nuchal folds, large carrying angles, 45 XO ? Dx?

A

Turner Syndrome - screen for cardiac probs with echo (coarctation, bicuspid aortic valve, MVP, hypo plastic heart), renal U/S (horseshoe kidney) , TSH 9hypothyroidism), and hearing test.

How well did you know this?
1
Not at all
2
3
4
5
Perfectly
166
Q

Work through diagnostics of delayed gastric emptying?

A

Could be from obstruction (internal/external) or impaired motility. R/o obstruction first with upper GI endoscopy, then consider CT ONLY if suspicious of external compression after non diagnostic endoscopy. Then consider scintigraphic gastric emptying study.

How well did you know this?
1
Not at all
2
3
4
5
Perfectly
167
Q

GI Motility agents?

A

Erythromycin and Metoclopromide.

How well did you know this?
1
Not at all
2
3
4
5
Perfectly
168
Q

Adolescents are at increased risk of what in pregnancy compared to those 20-24yrs?

A

perinatal mortality, preterm delivery, premature, and LBW infants

How well did you know this?
1
Not at all
2
3
4
5
Perfectly
169
Q

Indications for lung cancer screening?

A

Age 55-80 with 30 pack yr hx of smoking and still smoking OR recently suit within 15 yrs.

How well did you know this?
1
Not at all
2
3
4
5
Perfectly
170
Q

Increased incidence in spring/summer of skin infection with multiple, coalescing, small circular maculae that vary in color (white, pink, or brown); asymptomatic, seen on upper trunk, arm, neck, and abdomen = dx? Trmt?

A

Tinea Versicolor - organism prevents pigment tranfer to keratinocytes and makes affected skin paler than unaffected tanned skin.

Trmt = topical ketoconazole/terbinafine/clotrimazole/selenium sulfide

How well did you know this?
1
Not at all
2
3
4
5
Perfectly
171
Q

Seborrheic Dermatitis = looks like?

A

Inflammatory d/o with erythematous patches with oily scales, where there are many sebaceous glands

How well did you know this?
1
Not at all
2
3
4
5
Perfectly
172
Q

Glascow score

A

Intubate

How well did you know this?
1
Not at all
2
3
4
5
Perfectly
173
Q

Kids with fever, chills, flank/suprapubic pain = dx? Trmt considerations? Imaging?

A

Pyelonephritis. Oral cephalosporins are first line UNLESS patient is vomitting, failed oral abx before, or are hemodynamically unstable, or had positive blood culture.

If failing all antibiotics, renal U/S to assess for renal or perinephric abscess. Can also work up in kids

How well did you know this?
1
Not at all
2
3
4
5
Perfectly
174
Q

RF’s for constipation in children? Look for? Complications? Trmt?

A

RF’s = starting solid foods, cow’s milk, toilet training, school entry

Straining with BM, pellet stool, 2days s/p delivery

Complications = anal fissures, hemorrhoids, encopresis, UTI’s, vomiting, laxative, suppositories

Trmt - increased fiber, limit cows milk,.

How well did you know this?
1
Not at all
2
3
4
5
Perfectly
175
Q

Preexcitation syndrome w/ an accessory pathway resulting in a shortened PR interval (delta wave), widened QRS (> 0.12) = dx? Likely caused by? Trmt?

A

Wolff-Parkinson-White Syndrome - tachyarrhythmia. If symptomatic, catheter ablation recommended; controversial for those asymptomatic.

Atrial fib is especially dangerous because it can precipitate vfib in these patients.

How well did you know this?
1
Not at all
2
3
4
5
Perfectly
176
Q

MCC of SAH? Workup considerations?

A

Ruptured saccular aneurysm. Noncontrast Head CT = first choice. Lumbar puncture is required to exclude SAH.

+CSF = elevated openning pressure, xanthochromia, elevated RBCs in 4 tubes of CSF (if the RBC’s decrease in 4 subsequent tubes, it’s a traumatic tap)

How well did you know this?
1
Not at all
2
3
4
5
Perfectly
177
Q

Side effects of Nicotinic Acid?

A

Can lower triglyceride level but has Seffx: flushing, pruritus, and hepatotoxicity.

How well did you know this?
1
Not at all
2
3
4
5
Perfectly
178
Q

Rx’s for Triglycerides > 500 or

A

Fibrate (gemfibrozil) for >500; statin or Nicotnic Acid for

How well did you know this?
1
Not at all
2
3
4
5
Perfectly
179
Q

Who’s at risk for developing toxic megacolon early in their disease? Signs/symptoms? Mgmt considerations?

A

Those with IBD. 6cm dilated traverse colon, multiple air-fluid levels, loss of austral markings, pneumoperitoneum if perforated. Peritonitis signs may be absent!!! Patient look toxic (fever, tachycardia, “dry”).

Try medical mgmt first: fluids, abx, bowel rest, NG tube. Start steroids IF the underlying reason isn’t for infectious etiologies. AVOID ANTICHOLINERGICS AND OPIATES

How well did you know this?
1
Not at all
2
3
4
5
Perfectly
180
Q

Emotional/orthostatic stress (venipuncture, prolonged standing, heat exposure, exertion)

A

Vasovagal syncope. Clinical dx, only do tilt table if unclear dx.

How well did you know this?
1
Not at all
2
3
4
5
Perfectly
181
Q

Long term risks with PPI’s?

A

Osteoporosis/hip fracture (2/2 decreased Ca absorption, inhibits ocsteoclastic activity, reduces bone mineral density); higher risk of infections (C Diff), hypomagnesemia, interstitial nephritis, and decreased absorption of Vitamin B12 and Fe.

How well did you know this?
1
Not at all
2
3
4
5
Perfectly
182
Q

60% of head and neck cancers are locally advanced at time of diagnosis; what treatment modalities should be considered?

A

Combined chemo and radiotherapy.

How well did you know this?
1
Not at all
2
3
4
5
Perfectly
183
Q

List manifestations of Sarcoidosis. Pathophys of hypercalcemia?

A

Formation of non-caseating granulomas in lymph nodes and organs causes: bilateral hilar adenoapthy, interstitial;infiltrates, anterior uveitis, posterior uveitis, peripheral lymphadenopathy, hepatomegaly, splenomegaly, acute polyarthritis, central DI, hypercalcemia, maculopapular rash, nodular lesions, erythema nodosum

erythema nodosum, hilar adenoapthy, migratory polyarthralgias, fever = Lofgren’s Syndrome

Macrophages in sarcoid granulomas make 1-alpha-hydroxylase coverting 25-hydroxyvitamin D to 1, 25-dihydroxyvitamin D leading to increased absorption of Ca (see elevated Ca in urine with decreased serum PTH)

Give steroids to bring down Ca levels

How well did you know this?
1
Not at all
2
3
4
5
Perfectly
184
Q

Tremor presenting in upper distal extremities, more pronounced with outstretched arms, increases at end of activity = dx? rx? inheritance?

A

Familial tremor or benign essential tremor. Autosomal dominant inheritance. Can give beta-blockers

How well did you know this?
1
Not at all
2
3
4
5
Perfectly
185
Q

Acute PAINLESS monocular vision loss ddx?

A

Central Retinal Artery Occlusion - MCC = carotid atherosclerosis - lower intraocular pressure FAST with massage, anterior chamber paracentesis, IV acetazolamide, or mannitol. Cherry red spot and surrounding pallor

Central Retinal Vein Occlusion - likely non-embolic cause. Tortuous dilated veins, diffuse hemorrhages, disk swelling, and cotton wool spots. “flame storm”

How well did you know this?
1
Not at all
2
3
4
5
Perfectly
186
Q

Acute loss of vision with severe eye pain, injected conjunctiva, poorly reactive but dilated pupil, HA, N/V = dx?

A

acute angle-closure glaucoma

How well did you know this?
1
Not at all
2
3
4
5
Perfectly
187
Q

What are patients at risk for with TURP therapy?

A

Dry ejaculate 2/2 bladder neck failing to close after the procedure which enables sperm to flow backward to the bladder. (70% incidence)

How well did you know this?
1
Not at all
2
3
4
5
Perfectly
188
Q

Amiodarone’s effect on thyroid functions include:…

A

Can decrease peripheral conversion of T4 to T3 with minimal change in TSH = f/u

If it induces frank hypothyroidism, treat with levothyroxine

If it causes thyrotoxicosis, give steroids

How well did you know this?
1
Not at all
2
3
4
5
Perfectly
189
Q

RR and tidal volume correlate with = ? This is changed to regulate what ? Fi02 and PEEP are used to regulate what?

A

Ventilation - affects CO2
P02 - increasing PEEP can worsen hypotension by reducing preload and can recruit non-functioning alveoli with resultant barotrauma.

How well did you know this?
1
Not at all
2
3
4
5
Perfectly
190
Q

How do glucocorticoids cause bone loss and low Ca?

A

They decrease Ca gut absorption, cause renal Ca wasting, and have direct anti-anabolic effect on the bone. They suppress GnRH which can lead to hypogonadism and aggravate bone loss

How well did you know this?
1
Not at all
2
3
4
5
Perfectly
191
Q

Appearance of sporotrichosis? Trmt?

A

Ulcerates with non-purulent d/c and similar lesions forming along lymphatic tract. Trmt = itraconazole for 3-6months

How well did you know this?
1
Not at all
2
3
4
5
Perfectly
192
Q

Actinomysis RF’s?

A

Gingivitis, dental caries, extraction, and oromaxillofacial trauma. Chronic slow growing mass evolving into multiple abscesses, fistulae, and draining sinus tracts with thick yellow or serrous d/c

How well did you know this?
1
Not at all
2
3
4
5
Perfectly
193
Q

Tests to see if appropriate Rhogham given?

A

Rosette test = qualitative test to determine presence of feto-maternal hemorrhage. If negative, standard dose given. If positive, evaluate with Kleihaur-Betke stain or fetal red cell stain using flow cytometry

How well did you know this?
1
Not at all
2
3
4
5
Perfectly
194
Q

What’s the MCC of postoperative hypoxemia in post-op patient with recent thoracoabdominal sx occurring around 2-5 days?

A

Atelectasis - secondary to splinting, retained secretions, and diminished lung compliance. Obesity and OSA = RF’s

How well did you know this?
1
Not at all
2
3
4
5
Perfectly
195
Q

Differentiate nonallergic rhinitis and allergic rhinitis. Trmt?

A

NAR = nasal congestion, rhinorrhea, post nasal drip, no obvious trigger, develops later in life, erythematous nasal mucosa - intranasal glucocorticoids (fluticasone) or antihistamine (azelastine)

AR = watery rhinorrhea, sneezing, eye symptoms predominate, earlier age of onset, identifiable allergen, pal/bluish nasal mucosa, associated with other allergic d/o - intranasal glucocorticoids and antihistamines

How well did you know this?
1
Not at all
2
3
4
5
Perfectly
196
Q

Presentation for cardiac tamponade?

A

JVD, hypotension, diminished heart sounds, pulses paradoxus (drop in SBP > 10mmHg with inspiration), electrical alternans, right atrial and ventricular collapse during diastole

How well did you know this?
1
Not at all
2
3
4
5
Perfectly
197
Q

New onset DM, arthropathy in 2nd 3rd metacarpophalangeal joints, morning stiffness, decreased libido, hepatomegaly, subchondral cysts, sclerosis, osteopenia and hook like osteophytes on xray = dx? labs?

A

Hemachromatosis. Presents b/w 50-60 normally. Common for there to be CPPD crystals in joint. Get transferrin saturation, serum iron, and ferritin levels

How well did you know this?
1
Not at all
2
3
4
5
Perfectly
198
Q

Problem adducting the leg and sensory loss of small area in medial thigh?

A

Obturator neuropath - pelvic trauma or surgery

How well did you know this?
1
Not at all
2
3
4
5
Perfectly
199
Q

Inability to extend knee, loss of knee jerk reflexes, and sensory loss over anterior and medial aspects of the thigh, medial aspect of shin, and arch of foot = ?

A

Femoral nerve injury.

How well did you know this?
1
Not at all
2
3
4
5
Perfectly
200
Q

Problem with foot drop and weakness in foot dorsiflexion and eversion, paresthesias over dorsum of the foot and lateral shin = ?

A

Common personal nerve injury

How well did you know this?
1
Not at all
2
3
4
5
Perfectly
201
Q

Common complication of CABG, develops after pericardial effusion, develop peripheral edema, ascites, hepatic congestion with hepatomegaly, elevated JP, hepatojugular reflux, Kussmauls sign (lack of decrease in JVP with inspiration) and pericardial knock = dx? trmt?

A

constrictive pericarditis - antiinflammatory agents and pericardiectomy.

How well did you know this?
1
Not at all
2
3
4
5
Perfectly
202
Q

Modifiable RF’s for ischemic stroke?

A

HTN (most important), DM, smoking, and dyslipidemia

How well did you know this?
1
Not at all
2
3
4
5
Perfectly
203
Q

Common causative bugs in acute otitis media?

A

Strep Pna, H. Influe, and Moraxella Catarrhalis

How well did you know this?
1
Not at all
2
3
4
5
Perfectly
204
Q

Match it’s associated autoimmune d/o:

1) Ant-dsDNA
2) Anti-Centromere
3) Anti-Smith
4) Anti-Ro/SSA

A

1) SLE - anti dsDNA also helpful in following course of disease
2) CREST
3) SLE (less than dsDNA)
4) Sjogrens

How well did you know this?
1
Not at all
2
3
4
5
Perfectly
205
Q

Lupus treatment considerations:

A

Prednisone-cyclophosphamide = serious lupus manifestations (lupus nephritis, CNS or vasculitic manifestations)

Methotrexate-prednison = significant organ involvement with had incomplete response to prednisone alone

Hydroxychloroquine-prednisone = arthralgias, serositis, and cutaneous symptoms

How well did you know this?
1
Not at all
2
3
4
5
Perfectly
206
Q

Dermatitis herpetiformis = what presentation? association?

A

Associated with Celiac disease, and has pruritic vesicles and papules on elbows, knees, lower back, and buttocks

How well did you know this?
1
Not at all
2
3
4
5
Perfectly
207
Q

Human bites tend to have = infection type? Trmt?

A

Polymicrobial infections. Associated with alpha-hemolytic streptococci, step aureus, and anaerobic bacteria, Eikenella corrodes. Primary closure should be avoided. Start antibiotics and consider tetanus booster

Trmt = Oral amoxacillin/Clavulanate

How well did you know this?
1
Not at all
2
3
4
5
Perfectly
208
Q

Abnormal deposition of collagen in multiple organ systems associated with GERD, and Raynauds= dx? Dangerous effect on kidneys? Trmt for this complication?

A

Systemic Scleroderma - malignant htn can result!!! ACEi and IV nitroprusside

How well did you know this?
1
Not at all
2
3
4
5
Perfectly
209
Q

Meds for Strep Pharyngitis? Helps prevent ______ but not ______.

A

Oral PCN x 10 days; single IM PCN if cannot tolerate PO. 5 day course of Azithromycin is acceptable for PCN allergic patients

Prevents Rheumatic Fever but not post-strep glomerulonephritis (1-3wks s/p infxn)

How well did you know this?
1
Not at all
2
3
4
5
Perfectly
210
Q

Mesenteric ischemia MCC = ? from ?

A

Thrombus/emoblism from superior mesenteric artery

How well did you know this?
1
Not at all
2
3
4
5
Perfectly
211
Q

Papillary/follicular thyroid cancer surgery decision based on?

A

Neck U/S and LN biopsy - lobectomy for 1cm and/or tumor extension, multi-node involvement, mets

How well did you know this?
1
Not at all
2
3
4
5
Perfectly
212
Q

When to give joint injections for gout?

A

Contraindiciations to NSAIDS (bleeding, renal failure)

How well did you know this?
1
Not at all
2
3
4
5
Perfectly
213
Q

MCC’s of demyelinating ascending polyneuropathy = ? Monitor what?

A

Campylobacter jejuni, CMV, EBV, HSV. Monitor vital capacity!

How well did you know this?
1
Not at all
2
3
4
5
Perfectly
214
Q

ADPKD BP range and when to screen for berry aneurysms? MCC extra renal manifestation? How to screen asymptomatic family members?

A

BP

How well did you know this?
1
Not at all
2
3
4
5
Perfectly
215
Q

When to test for H Pylori?

A

Dyspepsia (abd fullness or pain w/o heartburn) - test if 55yo use endoscopy to r/o malignancy

How well did you know this?
1
Not at all
2
3
4
5
Perfectly
216
Q

Septate uterus has problem with _____ but not _____?

A

miscarriages, not implantation; abnormal blood flow to septate causes recurrent miscarriages. Treat with hysteroscopic metroplasty

How well did you know this?
1
Not at all
2
3
4
5
Perfectly
217
Q

Red, painful, sharply demarcated, edematous, elevated - MC on lower extremities but if seen on the face is butterfly distribution - has systemic symptoms like fever, chills, malaise - dx? Etiology?

A

Erysipelas secondary to Group-A Streptococcus

How well did you know this?
1
Not at all
2
3
4
5
Perfectly
218
Q

When to draw labs for lead poisoning? Treatment algorithm?

A

Test if: home built before 1978, peeling paint, pica, sibling with poisoning, low SES, immigrant. If level 70 = Dimercaprol + Ca disodium edentate (EDTA)

How well did you know this?
1
Not at all
2
3
4
5
Perfectly
219
Q

Which chromosomal abnormalities increase or don’t with AMA?

A

No increased risk = Turners; increased risk = Downs and Klinefelters

How well did you know this?
1
Not at all
2
3
4
5
Perfectly
220
Q

All children

A

> 7days. Renal and bladder U/S to evaluate for anatomical abnormalities. Voiding cystourethrogram is indicated in children with abnormal findings OR w/ recurrent UTI’s

How well did you know this?
1
Not at all
2
3
4
5
Perfectly
221
Q

Subacute/chronic knee pain, increased with squatting, prolonged sitting, using stairs = dx? What’s a test for this? Trmt?

A

Patellofemoral Syndrome - compress the patella and extend the knee. Exercise and strengthen thigh muscles = trmt

222
Q

Episodic pain and tenderness at inferior patella seen in jumpers = dx?

A

Patellar tendonitis

223
Q

Increased knee pain with sports, relieved with rest, tenderness and swelling at tibial tubercle = dx?

A

Osgood-Schlatter Disease

224
Q

Common cause medial knee pain with localized pain/tenderness distal to joint line - acute/episodic - dx?

A

Anserine bursitis

225
Q

Seen in people who work on their knees a lot, acute, highlight localized with visible swelling anterior of the patella- dx? Complication?

A

Prepatellar bursitis = housemaid’s knee. Complicated by secondary infection due to Stap Aureus

226
Q

Patient with neurologic symptoms who is having orthopnea, decreased ability to breathe while laying supine, paradoxical abdominal wall movement during inspiration, atelectasis on pulm exam = dx?

A

Diaphragmatic Paralysis

227
Q

Cerebral aneurysms causing suddent onset HA, nausea, nuchal rigidity, ptosis, anisocoria = SAH where?

A

Posterior communicating artery aneurysm w/ presentation due to location of CN III

228
Q

Childhood (

A

Respiratory Syncitial Virus = MCC of bronchiolitis

229
Q

URI causing hoarseness in children, barky cough, and inspiratory stridor from upper airway inflammation = dx? MCC?

A

Croup via Parainfluenza virus.

230
Q

Walking PNA bug?

A

Mycoplasma PNA

231
Q

Fatigue, exertional dyspnea, systolic murmur increased with valsalva, asymmetric septal hypertrophy = dx? Dominance? When to considered Implantable Cardioverter-Defibrillator?

A

Hypertrophic Cardiomyopathy = autosomal dominant.

Trmt = for those w/ symptoms, use negative inotropic agents (B-blockers, Verapamil, Disopyramide) - alcohol septal ablation is a last agent resort

ICD - those with hx of cardiac arrest or sustained ventricular tachycardia or at risk for malignant arrhythmias (family hx of sudden cardiac death, recurrent or exertional syncope, non sustained VT, hypotension w/ exercise, extreme LVH)

232
Q

Common drugs affecting Lithium levels = ?

A

Diuretics, NSAIDS (except aspirin), SSRI’s, ACEi’s, ARBS, antiepileptics (carbamazepine, phenytoin).

Use Beta-Blockers and Dihydropyridine CCB’s (Amlodipine) for BP control in patients with Lithium

233
Q

Warfarin can cause what to the fetus?

A

Nasal/limb hypoplasia and fetal bleeding. Can substitute with LMWH for most hypercoaguable patients (no crossing the placenta). However, if patient is extremely high risk, can use warfarin in 2nd/3rd trimester with substation of unfractionated heparin in last few weeks because it can be quickly reversed with protamine.

NO anticoagulation at onset of labor or before epidural

234
Q

What to give pts with alcoholic ketoacidosis?

A

IV saline with dextrose and thiamine (prevent Wernicke’s)

235
Q

Antibiotics most likely to cause seizure in those w/ hx, older age, renal insufficiency = ?

A

Beta lactams = PCNs, cephalosporins, monobactams, carbapenems, and fluoroquinolones

236
Q

Doxycycline side effect?

A

Bone discoloration and photosensitivity

237
Q

Gentamicin side effect?

A

Vertigo, ataxia, and ototoxicity

238
Q

Back pain associated with morning stiffness that gets better with exercise, insidious onset, lasts > 3 months, reduced range of forward flexion and reduced chest expansion = dx? How to diagnose? Monitor with? Most common and worrisome extraarticular manifestations?

A

Ankylosing Spondylitis - get an X-ray for DX. Monitor with AP/LA X-rays of lumbar spine and ESR

Associated with: acute anterior uveitis, aortic regurg, apical pulmonary fibrosis, IgA nephropathy, and restrictive lung disease, HLA-B27

Counsel against smoking!!!! No overall increased mortality - most have no functional or employment disabilities - encourage exercise!

239
Q

Eye redness and mucopurulent discharge = dx? Complications? Trmt? Common pathogens?

A

Bacterial conjunctivitis. Complication risk of keratitis (inflammation of cornea - foreign body sensation, photophobia, and corneal opacity/ulceration)

Trmt = Erythromycin ointment, sulfa drops, polymixin/trimethoprim drops for coverage of Staph aureus, strep pna, moraxella catarrhalis, and h flu.

Can return to social contact after 24 hrs s/p abx; ideally once no more d/c

240
Q

Fever, pharyngitis, and URI with injected conjunctiva = dx? Trmt?

A

Viral conjuncitivitis 2/2 adenovirus. Give Ciprofloxacin drops; also give these to contact wearers b/c risk of Pseudo!

241
Q

Primary amennorhea, breast development, no hair development, suspicious inguinal masses = dx?

A

Androgen Insensitivity - testosterone converted to estrogen so breasts develop but no mullein structures present! 46 XY 2/2 mutation of androgen receptor

242
Q

Difference b/w aspiration PNA and aspiration pneumonitis?

A

Aspiration PNA - infxn 2/2 gram + cocci, gram neg rods, and anaerobes in pts w/ RF’s occuring 1-5 days after event, w/ evidence of infection in dependent lung fields, treated with Clindamycin or Beta-lactam w/ beta-lactamase inhibitor

Aspiration Pneumonitis - inflammation of lung parenchyma due to aspiration of foreign contents (GI acid) occurring 2-5 hrs after pt with depressed LOC has event - can have cough, tachypnea, hypoxemia, respiratory distress, can see evidence of dependent lobe involvement - NO ABX

243
Q

Occurs 2/2 to renal ischemia, sepsis, nephrotoxins (aminoglycosides, radiocontrast media), FeNa >2%, muddy brown casts = dx? Urine Osmolality?

A

Acute Tubular Necrosis - approx. 300mOsm/kg vs. PreRenal Azotemia with BUN/Cr >20, FeNa 500 (trying to hold onto fluid so it’s concentrated)

244
Q

Fever, fatigue, young, pharyngitis, and posterior cervical lymphadenopathy = dx? Caused by? Caution against? Tests?

A

Infectious Mononucleosis by Epstein Barr Virus. Maculopapular Rash can develop if given Amoxicillin or Ampicillin = immune mediated by circulating antibodies to PCN derivatives

+ heterophile antibody test (monospot - can be negative in first week of illness); atypical lymphocytes, transient hepatitis

Avoid contact sports >3weeks

245
Q

Different manifestations of Lyme disease based on: acute (days - 1month), weeks to months, months to years after tic bite?

A

Days - 1 Month: Erythema Migrans, fatigue, malaise, HA, neck stiffness, myalgias, arthralgias

Weeks to Months: Carditis (AV block, cardiomyopathy), unilateral/bilateral CN involvements (CN VII), migratory arthralgias, conjunctivitis, multiple erythema migrans

Months - Years: Arthritis, encephalomyelitis, peripheral neuropathy

246
Q

Work up of Lyme if bite happened months ago?

A

Enzyme-linked immunosorbent assay followed by western blot. All IgG serologies will be + for Borrelia Burgdorferi

247
Q

What tests to order for chronic, symmetric polyarthritis with morning stiffness = ?

A

Rheumatoid factor and anti-citrullinated peptide antibodies

248
Q

Treatment of Lyme disease considerations?

A

28day course of oral doxycycline; or amoxicillin for children

249
Q

Treatment of acute seizure?

A

Benzodiazepines (lorazepam) > barbidurates (phenobarbital) or phenytoin/fosphenytoin

250
Q

Mental status changes, diaphoresis, tachycardia, HTN, hyperthermia, diarrhea, hydriasis, hyperreflexia, tremor, rigidity, myoclonus, ocular clonus = dx? Trmt? Cause?

A

Serotonin Syndrome. D/c rx’s, supportive measures, and sedation with BZD’s (severe cases need serotonin antagonist = cyproheptadine).

Not waiting >5wks before transitioning between SSRI’s (fluoxetine, paroxetine, sertraline, citalopram), MAOIs (phenelzine, tranylcypromine); or interaction with ondansetron, sumatriptans, St Johns Wort

251
Q

NMS differs from Serotonin syndrome by?

A

Has lead pipe muscular rigidity from dopamine antagonist.

252
Q

Sensory ataxia, lshooting pains, neurogenic incontinence, accommodating but non-reactive pupils = suspicious for?

A

Tabes Dorsalis 2/2 late neurosyphilis - differentiated from Vitamin B12 deficiency by spastic paresis and hyperreflexia.

253
Q

What can inhibit absorption of levothyroxine?

A

Calcium and Fe - take on empty stomach apart from other meds

254
Q

Management of calcified gallbladder?

A

Porcelain gallbladder is from chronic calcium salt deposits 2/2 chronic inflammation - high risk of GB carcinoma = surgery

255
Q

Seborrheic dermatitis causative agent? Trmt?

A

Malassezia furfur

Trmt = shampoos with ketoconazole, tar, zinc, pyrithione, or selenium sulfide

256
Q

Features of Histrionic personality d/o?

A

Excessive emoitionality, attention seeking behavior, provocative/seductive traits, considers relationships more intimate than they are.

257
Q

Antisocial d/o = ?

A

Disregard rules of others

258
Q

Borderline personality d/o = ?

A

Attention seeking, manipulative, exhibit self-injurious behavior, intense anger, chronic feelings of emptiness, splitting behavior

259
Q

Differentiate factors b/w alpha and beta thalassemia minor: ?

A

Both: low MCV, normal RDW (all small), target cells, normal or increased Fe and ferritin (2/2 more turnover), no improvement with response to Fe supplementation

Alpha: Normal electrophoresis

Beta: elevated Hemoglobin A2

260
Q

Medical vs. Surgical management for rectal prolapse?

A

Medical for non-full thickness prolapse - adequate fiber/fluid intake, pelvic floor exercises, biofeedback

Surgery for full thickness prolapse with concentric rings seems or for those with prolapse for fecal incontinence and/or constipation

261
Q

Treatment of HIV-thrombocytpenia?

A

Can occur at any time of the disease regardless of CD4 count or VL; distinguished by no confounding reason for thrombocytopenia. Giving the patient Antiretroviral Therapy generally improves it - only need to give corticosteroids or IV immunoglobulin if the patient is bleeding actively (this is transient fix)

262
Q

Expected VL drop with patients starting HAART for the first time?

A
263
Q

Infant 3-6 wks with post-prandial projectile vomiting, palpable olive shaped mass in abdomen = dx? Workup? RF’s?

A

Hypertrophic Pyloric Stenosis causing hypochloremic, hypokalemic, metabolic alkalosis. Get abdominal U/S (hint: get Abd Xray for Duodenal Atresia or malrotation)

RF’s = Postexposure prophylaxis for pertusis with erythromycin OR macrocodes in BF’ing women

264
Q

Causes of these on ECG:

1) Poor R wave progression
2) Prolonged QT

A

1) LVH, RVH, COPD, anterior infarct, conduction defects, and cardiomyopathy
2) Prolonged if measuring > R-R internal 2/2 anti arrhythmic drugs, antidepressants, hypokalemia, stroke, seizures

265
Q

Characteristics and trmt of Psoriatic Arthritis?

A

Pain and early a.m. stiffness relieved by physical activity in DIP and axial skeleton.

Those pts with psoriasis and joint pain or systemic manifestations warrant methotrexate.

IF pt does NOT have arthritis and only mild psoriasis (

266
Q

Who should you not treat asymptomatic bacteruria?

A

Nonpregnant, non menopausal women, elderly, diabetics, patients with spinal cord injury, patients with chronic indwelling catheters.

Treat pregnant women, those getting urologic interventions, and hip arthroplasty.

267
Q

Pt with elevated AFP should get what next before considering amniocentesis?

A

OB U/S

268
Q

Lesion of the skin that rapidly progresses into nodular patch with hemorrhage, ulceration, and necrosis = dx? Associated causative agent? Trmt?

A

Ecthyma Gangrenosum - Pseudomonas

Trmt = Aminoglycoside (tobramycin/amikacin) and extended-spectrum antipseudomonal PCN (piperacillin) or antipseudomonal cephalosporin (ceftazidime or cefepime)

269
Q

Prevent development of ventricular dysfunction from asymptomatic to symptomatic HF with?

A

ACE inhibitor. Consider digoxin to treat symptomatic HF

270
Q

Clues for patient with adrenal failure?

A

Patient’s w/ preexisting autoimmune d/o’s and weight loss, eosinophilia, low Na, high K, pre renal azotemia, and low blood glucose levels.

271
Q

What to advise patients taking valproate who are thinking of becoming pregnant?

A

High dose folic acid before conception. The use of anti epileptics is generally advised to continue their use EXCEPT valproate BUT you should only switch if she has not become pregnant yet!! If already conceived, do not switch.

NOT A CONTRAINDICATION FOR BF’ing

272
Q

Management of pts with bicuspid aortic valve? When to do surgery?

A

TTE for diagnosis. Screen first degree relatives (autosomal dominant with incomplete penetrance). Get an echo every 1-2 yrs to monitor for signs of: regurg, stenosis, ascending dissection, aortic root dilatation , endocarditis

Do surgery (balloon valvuloplasty) in symptomatic pts with AS, gradient > 50mmHg, and NO sig. AV calcification or regurg. Can also do in those planning to become pregnant

273
Q

PAtient not responding to antiresorptive bone meds, have very rapid progression of bone loss, multiple fractures, or have other constitutional symptoms (weight loss, pallor) = keep this on your DDX?

A

Multiple Myeloma - get an SPEP and UPEP

274
Q

Best non-invasive test for osteomyelitis? Gold std? MCC?

A

MRI. Bone Biopsy.

MCC = Staph aureus, GBS, Proteus, Pseudo, E Coli, Candida, Bacteroides, Peptococcus, and Clostridium. Get culture from deep curettage.

275
Q

What vision change can accompany viagra use?

A

Blue-green vision; thus pilots shouldn’t fly until after 6 hrs after taking it

276
Q

What can you give people with lactose intolerance to help with symptoms?

A

Yogurt with live culture

277
Q

In patients with suspected disseminated gonococcal infection, get what tests? Unique signs of this type of joint infection?

A

Culture the joint fluid, mucosal surfaces, urethra, cervix, rectal and oral mucosa

Unique = tenosynovitis at wrist, fingers, ankle, and toes. Pustular or vesicle-pustular skin rash that is transient and goes away in a few days

278
Q

Causes of hyponatremia?

A

Primary polydipsia, adrenal insufficiency, hypothyroidism, excessive ADH v. SIADH

279
Q

Causes of SIADH?

A

Small Cell Lung cancer, carbamazepine, cyclophosphamide, SSRI’s, pulmonary disease, HIV, post-operative patients (esp. when receiving large fluid boluses)

280
Q

Nausea, malaise, HA, lethargy, obtundation, seizures, coma, and respiratory arrest = course of hyponatremia - pathophy? trmt?

A

It’s from cerebral edema!!

Treat if Na falls

281
Q

How to differentiate etiologies of ascites?

A

SAAG =serum-ascites albumin gradient (their difference)

> 1.1 = portal htn: CHF, cirrhosis, alcoholic hepatitis

282
Q

Low TSH and low T4 should make you suspicious of? Look out for? Order?

A

Central hyothyroidism - look out for low sodium/glucose/hypotension. Can get serum ACTH and short ACTH stimulation test (cosyntropin, synthetic ACTH) to assess:

  • low response, low ACTH level = central adrenal insuff
  • low response, high ACTH = primary adrenal insuff

Should get MRI of the pituitary too.

283
Q

What to give patient who p/w: delirium, seizures, respiratory depression, sinus tach, hypotension, dry mouth, blurred vision, dilated pupils, flushing, hyperthermia w/ psych hx?

A

Sodium Bicarb = neutralizies TCA to non-iodized form making them less likely to bind to Na fast channels in the heart ,preventing QRS widening; if refractory give magnesium or lidocaine

284
Q

Causes of decreased medication use secondary to cytochrome P450 induction = ?

A

phenytoin, carbamazepine, ethosuximide, phenobarbital, topiramate

NOT valproate or gabapentin

285
Q

Atypical PNA’s = ?

A

PCP, Mycoplasma, Chlamydia

286
Q

Legionella PNA causes what symptoms?

A

Diarrhea, nausea, vomiting, HA, confusion - sputum shows many neutrophils, no to little organisms

287
Q

Area of hair loss w/o local scaling or inflammation = dx? trmt? prog? Ddx?

A

alopecia areata - discrete, smooth and circular area of hair loss over scalp - occurs over few weeks w/ recurring pattern; most have regrowth over time. Thought to be autoimmune (t-cell infiltrate near hair follicles - seen with other autoimmune d/o’s). Intralesional or topical steroids = trmt. Chronic recurring d/o.

Ddx: secondary syph (moth-eaten, scarring), tinea capitis (scaling, inflammation), discoid lupus (hypopigmentation, other lesions)

288
Q

ARDS assessed by? Pathophys?

A

PaO2/FiO2

289
Q

Mechanical vent settings in ARDS = ? Be careful of?

A

Low tidal volumes (8mL/kg then to 7 over next 1-3 hrs), plataeu pressure 95 can increase risk of toxicity (reactive O2 species) - decrease FiO2 once reaching goals

290
Q

Trmt for primary dysmenorrhea?

A

1st line = NSAIDS
2nd = OCPs
3rd = eval for secondary causes

Primary dysm 2/2 prostaglandin excess

291
Q

What surgery and when is it recommended in pts with carpal tunnel syndrome?

A

Volar carpal ligament release. If they have motor weakness or atrophy of thenar eminence.

Typical Trmt = splint > steroids > surgery

292
Q

Initial trmt for cancer of the glottis?

A

Radiation therapy, laser excision, or partial vocal cordectomy

293
Q

CPK levels in rhabdo? RF’s? Mgmt? Complications

A

CK > 10,000. RF’s= cocaine, severe trauma, exertion, NMS, hypothermia, hypothyroidism.

Mgmt = IV fluids and then alkalinize the urine

Complications = renal failure, hyperphosphatemia, hyperkalemia, hypocalcemia

294
Q

Criteria for MM? What should you get after fractures? Complications?

A

1) monoclonal protein in serum or urine
2) > 10% clonal plasma cells in bone marrow or soft tissue/bone
3) CRAB

Get skeletal survey to assess extent of fractures (punched out lytic lesions)

Complications = hypercalcemia (hydrate, steroids bisphosphonates), renal insufficiency 2/2 light chain cast nephropathy (plasmapharesis), Infection (vaccinate), hyperviscosity (bleeding, blurry vision, neuro changes - plasmapharesis), thrombosis

295
Q

Elevated IgM condition with associated risk of hyper viscosity?

A

Waldenstrom’s macroglobulinemia

296
Q

Treatment considerations for actinic keratosis? Risk?

A

Early lesions: 5-FU, imiquimod, cry destruction with liquid nitrogen

Biopsy if: unclear dx, lesion >1cm, ulceration present, tenderness, growing rapidly, refractory

Risk of SCC (1-20%)

Biopsy + SCC - get Mohs sx or excision with 4mm margins

297
Q

Absence of peristaltic waves in lower 2/3 of esophagus and decrease in LES tone w/ complaints of GERD and dysphagia (“sticking”) = dx?

A

Scleroderma

298
Q

Two mortality reducing settings for vent settings in ARDS setting?

A

Limited plateau pressure

299
Q

Orthopnea, paroxysmal nocturnal dyspnea, tachycardia, diffuse crackles, S3, JVD, peripheral edema = dx? Trmt considerations?

A

Decompensated Heart Failure.

Acutely give O2, IV loop diuretic (furosemide), consider IV nitroglycerin. If becomes hypotensive, give NE. Goal = reduce cardiac preload

300
Q

MC pathogen from cultures in corneal foreign bodies = ?

A

Coagulase negative Staphylococcus (other common = strep, Haemophilus, and Pseudo)

301
Q

CXray findings indicative of active TB = ?

A

Upper lobe infiltrates, cavities, hilar adenopathy, or pleural effusions

302
Q

when to treat children with dysentery with antibiotics?

A

Causing sepsis, if immunocompromised,

303
Q

Fever, crampy abdominal pain, low-volume bloody diarrhea in children = dx?

A

Bacterial Enteritis (dysentery) - Salmonella = MCC in US. Others = Shigella, E. Coli (O157:H7), Yersebia, Campylobacter - produce bloody diarrhea by directly invading enterocytes or injecting toxin into cells

*DONT GIVE ABX TO E COLI O157:H7 - PUTS AT RISK FOR HUS!

304
Q

Prog for patient with 1x episode of depression and treated successfully?

A

Stay on for 6months - 1 yr and then can be tried off meds. Once they’ve have 2 episodes, they should stay on!

305
Q

Confusion, ataxia, nystagmus = dx? Trmt? Complication?

A

Wernicke’s Encephalopathy. Give IV Thiamine

Korsakoff’s psychosis can endure with profound confabulation and anterograde/retrograde amnesia - see maxillary bodies and thalamus primarily affected in diencephalon

306
Q

Acute delirium/dementia in elderly patient with elevated MCV, reticulocytes, pancytopenia, increased indirect bili?

A

B12 Deficiency - don’t have to have motor signs!!!

307
Q

characteristics of MArfans?

A

Ectopia lentis, increased arm/height ratio, decreased upper/lower body ratio, arachnodactyly, breastbone dipping inward or outward, scoliosis/kyphosis, joint hyper mobility = auto dominant by mutations in ECM protein fibrillin-1.

Main M/M = aortic root disease (dilatation, AR, dissection) - all should get CT or echo. COUNSEL against intense sports

308
Q

Concurrent otitis media + purulent conjuncitivits = causative agent?

A

Nontypeable H flu

309
Q

AOM + nonpurulent conjunctivitis, URI, and gastroenteritis = causative agent?

A

H Influenzae

310
Q

Acute otitis externa = MCC?

A

Pseudomonas

311
Q

Trmt considerations for AOM?

A

Uncomplicated AOM = 10d high-dose amox. Repeat infxn within a month = Augmentin

312
Q

RF’s for contrast induced nephropathy? Prevent with?

A

Age > 75, reduced GFR, reduced renal perfusion (hypoten), increased contrast load

Prevent with IV saline or IV sodium bicarbonate - hold diuretics and NSAIDS

313
Q

Dysuria, dull/aching pain in perineal region and testicles, blood-tinged ejaculate = dx?

A

Prostatitis. Negative UA + >20 leuks/HPF on prostatic secretions = Chronic nonbacterial prostatitis (symptomatic treatment)

314
Q

Thick, brittle, discolored nails = dx? RF’s? Trmt?

A

Onychomycosis. RF’s = increased age, tine pedis, diabetes, PVD.

Dx w/ KOH, periodic acid-Schiff stain, culture

Trmt = terbinafine, itraconazole; second line = griseofulvin, fluconazole, ciclopirox (12wk therapy for toes, 6 wks for fingernails) . Trmt risks = hepatotoxicity, treatment failure, recurrence.

315
Q

Why CCB’s contraindicated in ACS?

A

Hypotension and subsequent increased risk of increased sympathetic reflex activity.

316
Q

Extraintestinal findings of Celiac’s Disease? How to screen?

A

Fatigue, wt loss, dermatitis herpetiformis, vitiligo, osteopoenia - malacia, anemia, peripheral neuropathy, HA, autoimmune thyroiditis, Type I DM, depression, psychosis

Screen with anti-endomysial Ab and anti-tissue transglutaminase Ab. Gold std = bx of Small Intestine

317
Q

Mets of prostate cancer locale? Best imaging? Trmt for mets?

A

Bones, especially axial skeleton.

Radionuclide bone scan.

Want androgen depletion: LHRH like leuprolide bind and cause LH/FSH release with transient rise in testosterone; dissipates after a week - this causes flare of symptoms, thus anti androgen (flutamide) is used 1 wk prior to block effects of elevated T

318
Q

Trmt of CIN 2 vs. 3 in young patient

A

Can wait and observe with pap/colpo with CIN 2 in prepregnant patient because of cervical stenosis and cervical insufficiency resultant from therapy. But if CIN 3, need LEEP, cryosurg

319
Q

Indications for carotid endarterectomy: ?

A

Men:

  • asymptomatic 60-99% stenosis
  • symptomatic 50-69% (IIA) / 70-99% (IA)

Women:
- symptomatic/asymptomatic 70-99% stenosis

320
Q

Back pain that gets better with sitting down, leaning forward/flexion, and gets worse with standing and extension = dx? trmt?

A

Lumbar stenosis - try conservative management, lumbar epidural block, or surgical decompression via laminectomy

321
Q

MRI findings w/ lumbar stenosis = ?

A

Encroaching osteophytes at the facet joints, hypertrophy of the ligamentum flavum, and protrusion of intervertebral disk in narrowing of spinal canal.

322
Q

When to give antibiotics in acute pancreatitis?

A

Contrast-enhanced CT showing 30% or more of the pancreas is necrotic , prophylactic abxs are of use (imipenem and meropenem)

323
Q

Marked increase in serum Cr s/p ACEi in renal transplant pts is suggestive of?

A

Renal artery stenosis

324
Q

Asymptomatic hyperkalemia with mild metabolic acidosis?

A

Hypoaldosteronism

325
Q

MCC of chronic adrenal insufficiency = ?

A

Autoimmune destruction.

Other = hemorrhagic infarction, metastatic disease, and infection (TB)

326
Q

Features of chronic adrenal insufficiency?

A

Hyponatremia, hyperkalemia, and hyperchloremia metabolic acidosis. = lab findings

Fatigue, weight loss, myalgia, increased pigmentation, and decreased axillary and pubic hair.

327
Q

A cause of sudden cardiac death that has no abnormal ECG or echo findings?

A

Anomalous origin of the coronary artery - left main artery from the right sinus of valsalva and the origin of RCA from left coronary sinus.

328
Q

Brugada syndrome = ?

A

RBBB and ST-segment elevations in leads V1-V3

329
Q

When to screen for chlamydia in women?

A

age 24 and less or those with high risk sexual practices

330
Q

Adolescents with upper-extremity myoclonic jerks and subsequent generalized tonic-clonic seizures = dx? Trmt? See on EEG?

A

Juvenile Myoclonic Epilepsy - Valproic acid. Bilateral polyspike and slow discharge in interictal periods

331
Q

When to get endoscopy with suspected C diff patient? Abx culprits?

A

If enzyme immunoassay is negative but high suspicion (neg lab tests), they aren’t getting better on abx, or need urgent dx (look for pseudomembranous colitis.

PCR = more sensitive than enzyme assay

Commonly associated abx = Fluoroquinolones, enhanced-spectrum PCNs, cephalosporins, and Clindamycin

332
Q

How to rapidly in tube someone?

A

Rapid sedative = etomidate, propofol, midazolam and paralytic = succinyl choline, rocuronium

333
Q

Evidence of PE on Echo?

A

RV strain = increased RV size, tricuspid regurg, decreased RV function, and RV thrombus

334
Q

Magnesium hydroxide is safe for longterm use?

A

Yes- it’s Milk of Mag

335
Q

Surgical treatment for RCC?

A

Confined in the renal capsule = Stage I (partial nephrectomy)

Extends through renal capsule but not through Gerota’s fascia (stage II) and radical nephrectomy is the choice (can do with stage III too)

336
Q

Cushing’s Triad indicating increased intracranial pressure?

A

Bradycardia, HTN, and respiratory depression

337
Q

Hyperthryoid symptoms, fever, painful, diffusely enlarged thyroid = dx? Trmt?

A

Subacute thyroiditis - inflammatory response causing release of stored hormones; has mixed inflammatory thyroid infiltrate of lymphocytes, neutrophils, histiocytes, and multinucleate giant cells; low radio iodide uptake b/c huge release of hormones already

Trmt = Bblocker and NSAID

338
Q

What abx penetrate prostate?

A

Fluoroquinolone (cipro or levi for 6-12 wks) or Bactrim

339
Q

Post partum endometritis abx? Biggest RF?

A

Clindamycin and gentamicin

Route of delivery - C/S!!!!

340
Q

Dog and cat bites get what?

A

Tetanus consideration! Amoxicilin/Clavulanate

341
Q

Low grade fever, lymphadenopathy w/ posterior cervical and occipital LN’s, and maculopapular rash beginning on face and spreading caudally, patchy erythema on soft palate= ? Complications = ?

A

Rubella

Acute arthritis, thrombocytopenia, and encephalitis.

Soft palate spots = forscheimer spots

342
Q

High fever that rapidly resolves, rosy nonpruritic rash originating on trunk spreading to extremities = dx?

A

Roseola

343
Q

Cough, Conjunctivits, and Coryza, fever, photophobia, blue-white Koplik spots, maculopapular rash (“brick red”) starting on face and spreads caudally to trunk/extremities = ?

A

Rubeola

344
Q

Screening for thyroid probs in preg with? What is the change of reference for total T4/T3?

A

TSH - then get total T4/total T3 if abnormal. Total T4/T3 reference range can be adjusted 1.5x

345
Q

Pain control for metastatic prostate cancer in axial skeleton that’s refractory to hormone therapy = ?

A

Focal external beam radiation therapy

346
Q

MC cancers to metastasize to the brain = ?

A

Lung, breast, melanoma, and colon cancer

347
Q

Signs on MRI of mets vs. primary brain tumors?

A

Multiple, well-circumscribed lesions, and large amount of vasogenic edema as compared to the size of the lesions

348
Q

What kind of test do you want to have a higher PPV? NPV?

A

Higher sensitivity = higher NPV (b/c lower FN)

Higher specificity = higher PPV (b/c lower FP)

349
Q

arrest of labor in first stage = criteria?

A

=/> 6cm, ROM:

  • no cervical change >4 hrs w/ adequate ctx
  • no cervical change >6hrs w/ inadequate ctx
350
Q

Good screening tools for sarcoidosis? Confirmation of dx?

A

Screening w/ ACE levels or CXray

Bx of palpable LN, subcutaneous nodule (NOT ERYTHEMA NODOSUM), enlarged parotid, lacrimal gland

351
Q

Acute ischemic colitis RF? Areas at risk?

A

Atherosclerosis = RF. Present w/ abdominal pain followed by bloody diarrhea.

Vulnerable areas = watershed areas (splenic flexure and rectosigmoid junction)

352
Q

Criteria for cervical insufficiency = ? RF’s? Trmt = ?

A

H/o painless dilation w/ second trimester loss, painless dilation in current pregnancy in 2nd trimester, h/o preterm birth and current CL

353
Q

Presence of multiple dilated bowel loops and absence of rectal air, no meconium passage, bilious emesis, visible transition zone b/w normal/narrow rectosigmoid and dilated descending colon = dx?

A

Hirschsprung Disease - absence of ganglion cells on rectal mucosal suction bx

Associated with congenital anomaly (renal) and chromosomal defect (Downs)

354
Q

What medication is associated with fluid retention in patient with heart failure?

A

Thiazolidinedione medications (Pioglitazone)

355
Q

Trmt choice for ALS?

A

Riluzole = glutamate inhibitor

356
Q

Abx trmt for acute rhinosinusitis?

A

First line = augmentin

2nd line = doxy or fluoroquinolones

357
Q

Complication of appendicitis in pregnancy depending on trimester?

A
1st = 1/3 can have SAB
2nd = PTD
3rd = pylephlebitis (infectious thrombosis of portal veins)
358
Q

Associated defects found with Down Syndrome children? Increased risk of?

A

Endocardial cushion defects (get echo), duodenal atresia, Hirschsprung’s, atlanto-axial instability, and hypothyroidism

Increased risks = Acute leukemia! Alzheimer dementia, autism, ADHD, depressive d/o, and seizure d/o

359
Q

Steps for workup of polycythemia?

A

Recheck Hgb. Get EPO level - majority cases caused by elevated EPO 2/2 chronic hypoxia from lung/cardiac disease. Elevated EPO could also indicated RCC. Low EPO = PCV primary, CML

JAK2 for additional testing for PCV

360
Q

Trmt regimens for latent tuberculosis (+ testing but negative CXR) = ?

A

Isoniazid + Rifapentine wkly for 3mo

Isoniazid mono therapy for 6-9 mo

Rifampin for 4mo

Isoniazid & rifampin for 4 mo

361
Q

Non-positional vertigo, dizziness, dysarthria, diplopia, and numbness = dx? RF’s? Causes?

A

Vertebrobasilar insufficiency 2/2 emboli, thrombi, or arterial dissection.

362
Q

Aortic valve area less than ____ = severe AS? Trmt of choice?

A

1.0 cm^2; onset of symptoms markedly affects prognosis in patients w/ AS

AV replacement associated wit harked reduction in symptoms and mortality in patients with symptomatic AS

363
Q

MAssive colonic bleeding in elderly: top 2 causes? Associated findings?

A

1) Diverticulosis

2) Angiodysplasia - associated w/ AS and ESRD

364
Q

Palivizumab for infants to prevent RSV = ? Associated prognosis?

A

Preterm birth

365
Q

Trmt for CAP?

A

Ceftriaxone and Azithromycin

366
Q

Abdominal pain, myalgia, sweating, drooling, watery eyes, confused, serious diarrhea, constricted pupils, salivating, wheezing = dx? Trmt?

A

Organophosphate poisoning. Give Atropine to reverse muscarinic effects and pralidoxime which activated cholinesterase’s.

367
Q

Acetominophen overdose = trmt?
Opiod intox?
BZD overdose?

A

N-acetylcysteine / Nalozone / Flumazenil

368
Q

When to do ORIF with clavicular fractures?

A

Open fractures, neurovascular compromise, skin tenting, widely displaced fractures, significant shortening.

If only minimally displaced or non displaced = conservative trmt

369
Q

Erection > 3hrs long = emergency; management algorithm = ?

A

Ice packs, alpha-adrenergic agonsit ( phenylephrine, epinephrine q5min) , oral terbutaline

370
Q

Signs of thyroiditis vs. other causes of hyperthyroidism?

A

Low radioactive iodine uptake (vs. general uptake in Graves vs. focal in nodular), elevated thyroglobulin (destruction of cells), no tenderness can help differentiate postpartum causes from subacute thyroiditis

371
Q

Chronic alcoholics are frequently depleted of this electrolyte even in the face of a normal serum level = ? Refeeding them can cause?

A

Phosphate. Refeeding can cause intracellular shift and lead to rhabdo.

372
Q

BF’ing concern with metronidazole?

A

If given for trick, give 2 g by mouth for 1 dose and BF’ing should be d/c’d for next 12-24hrs.

373
Q

What causes bleeding from diverticulosis?

A

Erosion of the artery 2/2 fecalith in diverticular sac

374
Q

RF’s for adenocarcinoma and SCC in the esophagus?

A

AC = chronic GERD / Barrett’s esophagus

SCC = EtOH and Tobacco

375
Q

Causes of Pseudotumor Cerebri = ?

A

Idiopathic, endocrine d/o’s (hypothyroidism, adrenal insufficiency, Cushing disease, hypoparathyroidism), Meds (Isotretinoin, tetracycline, steroids, danazol, tamoxifen, levothyroxin, lithium, nitrofurantoin)

376
Q

Chronic pancreatitis test = ? Advise?

A

CT - can show calcifications, enlargement, ductal dilatation, and pseudocysts

Of note, serum amylase and lipase may be slightly elevated or normal because they become decreased in pancreas as it’s fibrosed over time

Advise against EtOH and low fat diet, small frequent meals

377
Q

ECG patterns of MI’s = ?

A
Anterior = V1-V6
Lateral = I, aVR, aVL, V5-V6
Inferior = II, III, aVF
378
Q

PCOS trmt = ?

A

Weight loss > clomiphene if wanting to become pregnant

OCPs if not wanting to become pregnant

379
Q

Adjunct therapy for bipolar breakthrough?

A
  • FIRST, make sure Lithium level is therapeutic (0.6 - 1.2); closer to 1 is target
  • can add lamotrigine for depression
  • can add antipsychotic (risperidone) for severe mania/psychosis
380
Q

LLQ pain in older individual, +FOBT = dx? Outpt trmt?

A

Acute Diverticulosis

If stable on CT and clinical findings, can give Cipro and Metro for mild disease. May need CT to assess for abscess, perforation, and fistulas

381
Q

Handing solitary pulmonary nodule = ?

A

Up to 3cm, compare with previous X-rays - if none, get CT.

Benign features = serial CT scans

Indeterminate = Biopsy or PET

Suspicious = sx excision

382
Q

Trmt options for condyloma accuminata?

A

Internal = Trichloroacetic acid

External = TCA, Podophylin,

383
Q

Immune Thrombocytpenia = presentation? Labs? Trmt?

A

Can have previous viral infection, asymptomatic petechiae/ecchymosis, mucocutaneous bleeding. Likely due to anti platelet Ab

Isolated Thrombocytopenia 30,000 w/o bleeding = observe. Plt

384
Q

Features of DIC = ?

A

Leads to bleeding and clotting. Associated with sepsis, trauma, obstetric complications, malignant. Elevated D-dimer, prolonged PT, prolonged aPTT, low fibrinogen. = consumptive coagulopathy

385
Q

TTP characteristics = ?

A

thrombocytopenia, MAHA, elevated lactate dehydrogenase, renal failure, fever, and neuro symptoms

386
Q

Active bleeding variceal management = ?

A

Stabilize > early endoscopic intervention > repeat if necessary; if fails twice, consider TIPS or surgical shunting.

Once stabilized, can put on non-selective beta blockers (propanolol or nadolol)

387
Q

Level that warrants phototherapy in hyperbilirubinemia in neonates?

A

Serum bili > 18. If more than 25, that’s severe and are at risk for neuro complications.

388
Q

Unilateral thunderclap HA associated with Horner’s syndrome should consider = dx? Workup? Complications?

A

Carotid dissection. CT angio > MR angio or catheter angiography (gold std). Manage with antithrombotic therapy 2/2 stroke as a major complication

389
Q

Trmt of prolactinoma = ?

A

Cabergoline, bromocriptine = dopamine agonists

390
Q

Distortion of head, eyes, and involuntary tongue movements s/p medication initiation = dx? Trmt?

A

Acute Dystonic Reaction 2/2 antipsychotics, metoclopramide

Give IV Diphenhydramine. If refractory, give IV Benztropine

391
Q

Trmt for Salmonellosis = ?

A

None if immunocompetent and > 1yr old

392
Q

Trmt for post exposure HepB exposure in persons w/ and w/o adequate prophylaxis?

A

Hep B immune globulin and initiate vaccination within 12 hrs. Those previously immunized w/ adequate response = no trmt

393
Q

MC location of abdominal aneurysm? Screening? Mgmt? Repair?

A

Infrarenal (>3cm). Screen in men age 65-75 who have smoked. SMOKING CESSATION!!! Aspirin and statin therapy.

Repair: if large (>5.5), rapidly enlarging (>0.5cm in 6months), AAA w/ peripheral artery disease.

Can f/u

394
Q

Absolute contraindications for combined OCP’s?

A

Migraine w/ aura, >15cigarettes/day & > 35yrs old, BP > 160/100, H/o stroke/ischemic heart disease/VTE, cirrhosis,

395
Q

Granular casts, elevated WBC’s/HPF, 4+ proteinuria, and elevated Cr w/ h/o of NSAID used = dx??

A

Acute Interstitial Nephritis

396
Q

Small effusion, locking sensation with knee extension, can’t fully extend leg, joint line tenderness = dx?

A

Menicus injury; + McMurray test

397
Q

Common complication associated with compartment syndrome? Measure what for mgmt?

A

Acute Renal Failure - heme pigment released from damaged muscle is directly toxic to proximal tubular cells, combines with Tamm-Horsfall protein to form tubular casts, and induces vasoconstriction which reduces medullary blood flow

Measure tissue pressure (>30mm Hg OR DBP - compartment pressure

398
Q

Alternative med supplement that can cause HTN?

A

Licorice

399
Q

SLE, hematuria, proteinuria, renal insufficiency, HTN, marked hypocomplementemia = next work up step? Mgmt based on? Monitor what labs?

A

Renal Bx. Depends on type of glomerular involvement:
I/II - no trmt
III/IV - require immunsuprresion

Monitor Anti-dsDNA and complement levels

400
Q

Workup of testicular mass?

A

Transillumination test > U/S; if suspicious for malignancy, follow w/ CT and measure tumor markers (B-hcg, alpha fetoprotein); if high suspicion, gold std = radical inguinal orchiectomy

401
Q

First step in evaluating ASYMPTOMATIC person w/ exposure to TB?

A

PPD

402
Q

Endometriosis mgmt?

A

Can try emperic therapy w/ NSAIDS, GnRH analogs, Danazol - synthetic androgen, OCP’s

BUT, if the patient is expediency severe symptoms, infertile, and unresponsive to conservative pain meds, get laparoscopy first to confirm diagnosis. May need surgical therapy

403
Q

If you have elevated TSH, T3/T4 = ddx? Differentiate?

A

TSH-secreting pituitary adenoma or thyroid resistance syndrome; the former has elevated alpha-subunit levels; the latter normally has hypothyroid clinical picture

404
Q

What is familial dysalbuminemic hyperthyroxinemia?

A

Abnormal albumin has high affinity for T4 but not T3. Normal TSH

405
Q

Densley pigmented lesion with irregular borders seen on optho exam = dx? Mgmt?

A

Ocular melanoma = primary malignant tumor from melanocytes.

U/S = most sensitive to diagnose the lesion while MRI used to document any extrascleral extensions for staging/trmt purposes

If asymptomatic and small (

406
Q

Squamous cell skin cancer trmt option?

A

Surgical excision!!

Also can use cry therapy, electrosurgery, and radiation therapy.

407
Q

Trmt options for Grave’s hyperthyroidism = ?

A

Antithyroid drugs = mild cases, older, pregnant

Radioactive Iodine = mod/severe disease w/ or w/o opthalmopathy

Thyroidectomy = very large goiter, suspicion for cancer, coexisting parathyroidism, pregnant patients unable to tolerate antithyroid drugs, severe opthalmopathy, obstructive symptoms

408
Q

What to measure s/p radioactive iodine in Grave’s patients for eval?

A

Get Total T3 and free T4 to monitor response.

thyroid antibodies can help predict relapse in patient with treatment but doesn’t gauge functional status

409
Q

When to use amiodarone in ACLS?

A

Wide complex, regular tachycardia - pulseless VT/VF refractory to initial defib

410
Q

All patients with known atherosclerotic CVD regardless of baseline LDL levels should have this onboard to improve cardiovascular outcomes and mortality?

A

Statins!

411
Q

Light’s criteria?

A

Transudate = pleural/serum protein 50%, LDH > 60% or > 2/3 ULN

412
Q

UTI rx’s safe in preg?

A

Amoxicillin, cephalexin, or Nitrofurantoin

413
Q

PE/symptoms that should prompt you to think of Sjogren’s? Increased risk of what malignancy?

A

Keratoconjunctivitis sicca (gritty, burning sensation), Xerostomia (dry mouth, many dental caries) - get Anti-Ro/SSA and Anti-La/SSB.

Schirmer test confirms eye problem.

B-cell non-Hodgkin’s lymphoma 2/2 polyclonal B cell activation and infiltration of the salivary glands

414
Q

Erythemaouts, itchy, scaly painful rash which starts cleaning from the center and can migrate + diarrhea + DM + angular cheilosis +facial flushing = dx?

A

Glucagonoma. Necrolytic migratory erythema. Most are malignant and mainly mets in liver. Also have VIP, calitonin, and GLP1 release

415
Q

Dementia, diarrhea, dermatitis, stomatitis, and cheilosis = dx?

A

Pellagra from niacin deficiency

416
Q

Type I vs. II error

A

Type II = fail to reject a null hypothesis when it’s false (false negative); w/ higher sample size, lower Beta, less chance of type II error

Type I = rejects a null hypothesis that is true (false positive)

Power = 1 - B (type II error)

417
Q

Correlation coefficient info

A

-1 to +1, 0 being no correlation (if negative, one goes up while the other goes down)

418
Q

Factitious d/o = ?.

A

When a pt inflicts harm on themselves to act the sick role. Differs from malingering b/c no external incentive

419
Q

Likelihood ratio = ?

A

Does not vary with prevalence of disease = probability of a given test result in a pt with d/o compared to probability of same result in patient w/o d/o

Positive = sens / (1-spec)

Neg = (1-sens) / spec

420
Q

Top reasons for Fever of Unknown Origin?

A

Connective tissue disease, infection, malignancy

Dx of exclusion with fever on multiple occasions over a time space exceeding three weeks and s/p negative workup

421
Q

Risks to infants of Diabetic mothers?

A

Macrosomia, hypocalcemia, hypoglycemia, hyperviscosity 2/2 polycythemia, respiratory difficulties, cardiomyopathy (hypertrophic interventricular septum), CHF

422
Q

Indications for parathyroidectomy in primary PTH?

A

All symptomatic patients. Asymptomatic patients with serum Ca > 1 above ULN, young age (

423
Q

Dull crampy abdominal pain worse after meals in a vasculopath w/ negative work up = ? Workup?

A

Chronic mesenteric ischemia. CT/MR angio, duplex U/S = non-invasive methods for evaluating and locating obstruction

Angiography = gold std

424
Q

Child with sore throat, trouble swallowing, muffled voice, tongue hanging out sometimes, drooling, difficulty breathing = causative agent?

A

Haemophilus influenzae b causing epiglottitis (can see “thumb print” sign)

425
Q

N/V/D, increased bowel sounds, increased HR/BP, can’t sleep, myalgia, arthralgia, lacrimation, rhinorrhea, piloerection, mydriasis = dx? Trmt?

A

Opiod withdrawal.

Agonists = Methadone or buprenorphine

Nonopiods = clonidine, BZD’s, antiemetics, antidiarrheals

Depends on treatment center - if the patient can be monitored closely, can give agonist (2/2 worsening from partial agonistic effect)

426
Q

How to transport severed digit?

A

Wrap in sterile gauze, dampen with saline, put in sterile bag. Put in cooler w/ 50% water 50% ice

427
Q

Trmt prophylaxis for those in close contact with meningococal infection?

A

Rifampin (600mg BID x 2days)

Ceftriaxone (250mg once)

Ciprofloxacin (500mg once)

428
Q

Mgmt algorithm of epistaxis?

A

Nostril pinching > topical vasoconstrictor (Oxymetazoline) > Silver nitrate w/ topical lidocaine > anterior nasal packing with bacitracin

429
Q

Matrix glycosaminoglycan accumulation (myxedema, macroglossia, coarse dry skin), depression, myalgia, hypercholesterolemia, delayed DTR’s, cognitive slowing = screen for?

A

Hypothyroidism - can have macrocytosis, hyponatremia, and dementia

430
Q

> 50, pain or decreased ROM in neck/shoulder/hips, constitutional symptoms, elevated ESR, bilateral pain and morning stifness = dx? Associated with?

A

Polymyalgia rheumatica (differs from RA b/c no synovitis) - patient points to soft tissue as painful, not joints. Associated with Giant Cell Arteritis!!! Glucocorticoids = treatment of choice.

431
Q

When do you not have to taper glucocorticoid therapy?

A
432
Q

What is Immune Reconstitution Inflammatory Syndrome?

A

Paradoxical worsening of preexisting infections in HIV + individuals that occurs days-weeks after initiation of trmt with HAART. Best managed w/ HAART and continued abx for underlying pathogen

433
Q

In addition to surgery, what should you give to someone with perforated PUD?

A

IV fluids, IV PPI, abx

434
Q

Right colon and cecum dilatation w/ no evidence of mechanical cause = ?

A

Ogilvie’s Syndrome

435
Q

Primary CNS lymphoma = associated with? Related to what causative bug? Trmt?

A

HIV 2/2 EBV. Give HAART, radiation therapy, corticosteroids

436
Q

What are the pertinent lab values for pseudohypothyroidism?

A

Hypocalcemia, hyperphosphatemia - it’s secondary to resistance of PTH on its target tissue VS. Vit D Deficiency = low Ca and Phosph

437
Q

What type of cells are seen in diffuse alveolar hemorrhage syndromes?

A

Hemosiderin laden macrophages; in Wegener’s, Goodpasture’s and other systemic vasculitis

438
Q

Best diagnostic vs. best next step in mgmt for someone with suspected endobronchial obstruction?

A

Best initial mgmt = CT

Most diagnostic = flexible bronchoscopy

439
Q

Management of hypocaliuric hypercalcemia = ?

A

Nothin - Familial Hypocalicuric Hypercalcemia (problem with Ca-sensing receptor)= PTH elevated but WNL, elevated Calcium, urine calcium

440
Q

Plasma osmolality = ?

A

2xNa + Glucose/18 + BUN/2.8

NL = 280-290

441
Q

Correcting hyponatremia = ?

A

If mild/moderate (115-124) can be managed with H2O restriction. If there’s an identifiable cause, treat underlying cause

Severe

442
Q

Signs of ethylene glycol (antifreeze) intoxication? Trmt?

A

Metabolized by alcohol dehydrogenase into oxalic acid. See severe anion gap acidosis, Kussmaul’s respiration (rapid/deep)

Antidote = Fomepizole (competitive inhibitor of ADH)

443
Q

Remove IUD in asymptomatic female w/ IUD and + actinomyces?

A

No - this gram-positive anaerobic bacillus = common in normal vaginal flora w/ low risk of complication.

444
Q

Congenital rubella features = ?

A

Sensorineural hearing loss cardiac anomalies, cataracts, glaucoma

445
Q

How to differentiate volume depletion vs. hepatorenal syndrome?

A

Patients with cirrhosis have decreased vascular resistance 2/2 to splanchnic vasodilatation which would not respond with a fluid bolus (since both have BUN/Cr > 20:1) - if diagnosed should give octrotide, midodrine and albumin ( or NE by itself)

446
Q

Nystagmus in awake or agitated patients is characteristic of = ? trmt?

A

PCP intoxication (difference with other intoxications is that pts are sedated) - put in low-stimulation environment

447
Q

Lyme trmt if there is carditis or neuro symptoms other than cranial nerve palsy?

A

IV ceftriaxone

Normal Trmt = doxy, amoxicillin, or cefuroxime

448
Q

Tomatoe-red plaques and satellite papules in diaper area in setting of abx use = dx? Trmt?

A

Candidal diaper rash. Differs from diaper dermatitis because it does NOT spare crural folds like diaper rash does.

Trmt = antimycotic cream (clotrimazole)

449
Q

In patient w/ extensive smoking hx, recurrent PNA in same place or not resolving = suspicious for?

A

Obstructing endobronchial malignancy

450
Q

Rabies vaccination criteria = ?

A

Pre-exposure prophylaxis = rabies vaccine

Post-Exposure Prophylaxis (not vaccinated before) = rabies immunoglobulin and rabies vaccine

Post exposure w/ previous vaccination = rabies vaccine

451
Q

Only conditions you’d get a pap smear before 21?

A

Immunocompromised (HIV, SLE, organ transplants) at the time of sexual intercourse

452
Q

Trmt of PNA in patient’s w/ Cystic Fibrosis? MCC = Staph when young, Pseudo when older

A

Cefepime and amikacin - want pseudomonal coverage! S. Aureus if another offender! Add vancomycin if had MRSA before

453
Q

Diagnosis of rheumatic fever?

A

JONES Criteria = Joint pain (migratory), carditis, subcutaneous nodules, erythema marginatum, sydenham chorea.

Late findings = Mitral regurg/stenosis

Prevent with PCN for Group A Strep (Strep. Pyogenes)

454
Q

Lower extremity claudication, absent or dimished femoral pulses, erectile dysfunction = dx? Evaluate what?

A

Leriche Syndrome

Evaluate ABI -

455
Q

Treating Lead intox guidelines = ?

A

Mild (5-44) = no rx’s

Mod (45-69) = Meso-2,3-dimercaptosuccinic acid (DMSA, Succimer)

> 70 = Dimercaprol + Calcium Disodium Edetate (EDTA)

456
Q

Stages of euthyroid sick syndrome =

A

When they’re ill = low T3, T4, and TSH.

Then low T3, normal T4 and TSH levels = MC findings

Recovery phase = slightly elevated TSH, normal T4

No trmt necessary!!!

457
Q

Fever + vesicles and ulcers on anterior oral mucosa and around mouth = dx?

A

Herpes gingivostomatits 2/2 HSV1

458
Q

Intention to treat analysis helps support what study quality?

A

Randomization

459
Q

Screening protocol for those with increased CRC risk w/ first degree relative

A

Colonoscopy at 40 or 10 yrs before fam dx (WHICHEVER IS SOONER) and repeat every 3-5 yrs

*Not needed if fam member dx’d > 60 yrs

460
Q

Common agents able to cause acute pancreatitis?

A

Diuretics, IBD rx’s (5-ASA), Immunosupressive agents, Seizures/Bipolar use of Valproic acid, AIDS (Didanosine, Pentamidine), Abx (metronidazole, tetracycline)

461
Q

CHA2DS-VAS2C Score = ?

A

CHF, Hypertension, Age >75 (2pts) or Age 65-75 (1pt), DM, Stroke/TIA, Vasculopathy (prior MI), Sex (female)

Anticoagulate with AF and score 2 or more

462
Q

Target-specific oral anticoagulants are approved for what patients with AF?

A

Those w/ non-valvular AF (dabigatran, rivaroxaban, apixaban, edoxaban)

DON’T use w/: MS, prosthetic heart valves, ESRD, severe decompensated valvular disease

463
Q

SLE pts at risk for development of what that subsequently leads to leading cause of mortality in pts w/ SLE?

A

Premature Coronary atherosclerosis and accelerated atherosclerosis

464
Q

Neonatal chlamydial infections can cause what two things?

A

Conjunctivitis - occurs 5-14 days, thickened injected conjunctivae, water/muco/blood-stained discharge, eyelid swelling, pseudomembrane on eyes

PNA - 4-12wks, staccato cough, nasal congestion, hyperinflation

TRMT = oral erythromycin for 14 days

465
Q

Dx choice and mgmt for acute mesenteric ischemia?

A

CT angio of abdomen. Fluid resuscitation, abx, correct metabolic acidosis, NG tube for decompression, sx consult

466
Q

Doxazosin = what type of blocker? Consideration with Sildenafil?

A

Alpha blocker - give 6 hrs apart from Sildenafil to reduce risk of hypotension

467
Q

Hydrophobia, aerophobia, pharyngeal spasms, agitation, ascending paralysis, hx of spelunking w/ no bat scracth/bitch recalled = dx? Prognosis?

A

Rabies - can be from unrecognized contact or aerosolized particles in caves

Post-exposure prohylaxis = effective IF given PRIOR to manifestation of symptoms. Mst symptomatic pts develop coma and death within weeks

468
Q

2 contraindications for Tdap?

A

anaphylaxis within 7 days of administration of previous Tdap

encephalopathy within 7 days of Tdap

469
Q

Lumbosacral myelomeningocele is associated with what MC extra neural finding?

A

Bladder dysfunction > GI probs > lower extrem fractures

470
Q

Red flags for back pain = ?

A

Age >50, nighttime pain, constitutional symptoms, hx of malignancy, trauma, IVDU, infectious risk, >1month w/o improvement

471
Q

Differentiate Statin-induced myopathy with PMR?

A

statin-induced has elevated CPK and normal ESR vs. elevated ESR and normal CPK in PMR

472
Q

Features of Tuberous Sclerosis = ? Screen for? Leading cause of death?

A

AD or de novo mutation.

Derm - hypopigmented macules

Neuro - Epilepsy, cognitive disabilities, autism (progressive seizures= MCC of death)

CV - rhabdomyomas

Renal - Angiomyolipomas

Screen for: skin exam, fundoscopy, brain MRI, EEG, abdominal U/S

473
Q

Nelson’s dx = what two features?

A

Microadenoma with suprasellar extension on MRI and extremely high plasma ACTH levels

474
Q

Symptomatic pregnant patients w/ BV = trmt options(2)?

A

Oral metronidazole (crosses placenta, no known teratogenicity) or oral Clindamycin

475
Q

C. Diff Trmt algorithm ?

A

Mild-Mod (WBC 15k, Cr > 1.5x, Serum albumin 20k, toxic megacolon present; then subtotal colectomy, diverting loop ileostomy

476
Q

Trmt for reccurent C dif??

A

First recurrence = same as initial therapy

Second = pulsed oral vancomycin for 6-7 wks

More = fecal transplant

477
Q

Non-occupational HIV post exposure prophylaxis = ?

A

High risk exposure via high risk contact - begin triple therapy within 72 hrs for 4wks! Don’t initiate if >72hrs

478
Q

Arthralgias, palpable purpura, lymphadenopathy, HSM, peripheral neuropathy, asymptomatic hematuria/proteinuria/mild Cr elevation = dx? Historical clue or RF for patient = ? What Complement will be low? Trmt?

A

Mixed Cryoglobulinemia. Hep C!!!

Low C3/C4/CH50

Get cryoglobulin level!

Trmt = alpha-interferon and ribavirin (if no renal dysfxn); only alpha-interferon if sig renal impairment

479
Q

Granulomatosis w/ polyangitis = test?

A

Upper airway and kidney involvement!

C-ANCA; has normal complement levels

480
Q

ASO used for what screening? Affected Complement?

A

Post-streptococcal glomerulonephritis

C3/CH50 = low, normal C4

481
Q

Attributable risk percent = excess risk that estimates the proportion of disease among exposed individuals = calc?

A

ARP = (RR - 1)/RR

OR

ARP = (risk of exposed - risk unexposed)/risk of exposed

482
Q

Population attributable risk percent = excess risk in total population, not only exposed subjects = calc?

A

PARP = (Risk in total Pop - Risk in Unexposed)/Risk in Total Pop

Can use weighted average to calculate Risk in total Pop= (risk in smokers)(proportion of smokers) + (risk in unexposed)(proportion of nonsmokers)

OR

PARP = (Prevalence)(RR-1) / [(Prevalence)(RR - 1) +1]

483
Q

Who to give PPSV23 and PCV13 pneumococcal immunization to?

A

Give 23 for common comorbidities in those 19-64

Sequential 13 and 23 for those at high risk (immunocompromised) in same age group

Sequential in those >65

484
Q

Mitral Stenosis = sound?

A

Low-pitched rumble at the cardiac apex

485
Q

Be wary of this side effect of blood transfusions in those with kidney/liver/hypothermia/shock or those in surgery needing transfusions =?

A

Hypocalcemia secondary to these patient’s poor metabolism of citrate, which binds to calcium but might not show on measured serum Ca

486
Q

Signs of hypocalcemia = ?

A

Hyperactive DTR, muscle spasms in the face upper extremities, diaphoretic, bilateral hand contracture, tonic clonic seizures

487
Q

Anatomical predisposing factor for MAllory-Weiss Syndrome?

A

Hiatal Hernia

488
Q

When to start PCP prophylaxis with HIV?

A

CD4

489
Q

When to start prophylaxis against MAC?

A

CD4

490
Q

How to evaluate patients on cardiotoxic chemo?

A

Radionuclide ventriculography - decrease in EF >10% may warrant discontinuation

491
Q

Mgmt of hepatic adenomas on OCP’s?

A

D/C OCP’s. If >5cm, evaluate for surgical ressection OR if symptomatic.

2 risks = rupture or malignant transformation

492
Q

Carbamazepine risks involving blood lines and fluid levels?

A

Can cause marrow suppression = fever, mouth ulcers, easy bruising

SIADH

493
Q

Profile description for traumatic tap?

A

RBC > 6k, elevated WBC (1per 750-1000 RBCS), elevated protein and elevated glucose

494
Q

Side effects of Haloperidol?

A

Sedation, orthostatic hypotension, Anticholinergic side effects, tar dive dyskinesia, extrapyramidal symptoms (can add on BZD as adjunct to reduce extrapyramidal side effects)

495
Q

Children with Failure to Thrive have associated ___ at home.

A

Defined as weight less than 5% in those

496
Q

Complications of lead poisoning in children?

A

Cognitive impairment, behavior problems, encephalopathy, constipation, abdominal pain, decreased Vit D absorption, Interstitial nephritis, Hemolytic anemia

497
Q

General acne mgmt principles?

A

Start with topical retinoids and benzyl peroxide > topical abx (erythromycin, clindamycin) > oral abx (tetracyclines)

498
Q

Beside IV diuretics in those with acute decompensated HF, what else can you give patients to help?

A

IV vasodilators (nitroglycerin, nitroprusside)

499
Q

Methods for decreasing BP in patients with aortic dissection?

A

Want to control BP with IV Beta-blockers like labetalol, propanolol, or esmolol to SBP

500
Q

How to differentiate two main causes of precocious puberty?

A

Precocious puberty = premature activation of HPOG axis; tend to see sequential and gradual development of testicular enlargement, penis enlargement, pubic hair growth, and growth spurt

VS.

Precocious Pseudo Puberty = gonadotropin-independent process (excess sex steroids) - p/w severe androgen excess (severe acne, significant growth acceleration)

501
Q

Differences b/w Ehlers Danlos and MArfans?

A

ED - transparent skin, hyper extensible skin, easy bruising, velvety (doughy) fragile skin w/ atrophy, pectus excavatum, arched palate, scoliosis, MVP, develop hernias, COL5A1/COL5A2 gene mutations, AD

Marfans - pectus carinatum, spontaneous PNTX, FBN1 gene mutation, AD

502
Q

MC in Mexico, Central/S. America, biventricular heart failure w/ cardiomegaly, ventricular apical aneurysm, mural thrombosis w/ embolic complications, fibrosis leading to conduction abnormalities, progressive dilation of esophagus and colon = dx?

A

Chagas Disease

503
Q

Mgmt of inguinal hernias in pediatric group?

A

Should be surgically repaired early as possible 2/2 complications like incarceration esp. if it remains open during first months of life

504
Q

_____ is a major cause of respiratory muscle weakness and can lead to the failure of being able to wean patients off vent; 2/2 continuous glucose infusions (esp. alcoholics)

A

Hypophosphatemia

505
Q

Causes of Zinc deficiency?

A

Diarrhea, diuresis, malnutrition, CRF, burns and prolonged illness. More susceptible to infections and skin rash.

506
Q

Nonimmune direct effect of heparin on platelet activation and presents in first 2 days of heparin exposure = dx?

A

Type 1 HIT - continure Heparin as platelets will normalize

Can see necrotic skin lesions

507
Q

Immune-mediated d/o 2/2 Ab’s to platelet factor 4 complexed with heparin; leads to platelet aggregation, thrombocytopenia, and thrombosis (arterial and venous); presents 5-10d after heparin initiation = dx?

A

Type 2 HIT

508
Q

Mgmt s/p suspecting HIT?

A

STOP ALL HEPARIN!!! Confirm w/ serotonin release assay, heparin-induced plt aggregation assay, heparin-PF4 antibody ELISA

If + and needs anticoagulation, use direct thrombin inhibitor

509
Q

Drugs for smoking cessation?

A

Nicotine patches - no significant side effects; helps with cravings and withdrawal symptoms

Bupropion - decreased post-cessation wt gain, good choice in depressed pts - DONT USE IN SEIZURE OF EATING d/o

Varenicline = more effective than bupropion or NRT- HIGHER RISK OF CARD EVENTS, INCREASED RISK OF SUICIDE AND DEPRESSION

510
Q

Weight loss, splenomegaly, marked WBC count, + Philadelphia chromosome (translocated 9/22) creates bcr/abl fusion protein causing unregulated tyrosine kinase activity = dx? Trmt>?

A

Chronic Myeloid Leukemia

Tyrosine Kinase Inhibitors (Imatinib)

511
Q

Dx if pt p/w old age/critically ill, fever leukocytosis, and steady RUQ pain radiating to back and R shoulder but no gallstones on abd U/S = dx? If unsure get?

A

Acalculous acute cholycystitis = higher M&M

Get HIDA if unsure

512
Q

Etio of cat-scratch fever? Complications?

A

Bartonella Henselae

Complications = lymphatic suppuration, neuroretinitis, encephalopathy, HSM

513
Q

Abx for pyelo in pregnancy?

A

Ceftriaxone or Amp/Gent until afebrile for 24hrs - entire PO abx for 10-14 days

514
Q

Trmt for metastatic brain disease = ?

A

Whole brain radiation therapy can improve survival 3-6months

515
Q

Filaggrin problem (skin barrier), associated with intense pruritic skin lesions that have lichenification, flexural eczema, high serum IgE, Eosinophilia, increased leukocyte phosphodiesterase = dx? Trmt?

A

Atopic dermatitis

Avoid hot/dry climates, harsh soaps and detergents. Regular use of emollients to help skin hydration + oral antihistamines are helpful!!

If refractory: topical glucocorticoid cremes (hydrocortisone)> high potency = triamcinolone, betamethasone (BUT DONT USE ON FACE…use calcineurin inhibitors = Tacrolimus)> UV light

516
Q

Mgmt of cocaine-induced CP?

A

DONT GIVE B-BLOCKER! Give lorazepam to decrease anxiety and agitation. If refractory BP, use phentolamine (alpha antagonist), nitroglycerine, or nitroprusside

517
Q

ADPKD extra-renal manifestations = ?

A

Hepatic/pancreatic/splenic. and pulm cysts; cerebral aneursyms, aortic aneurysm, colonic diverticula, MVP, inguinal/abdominal hernia

518
Q

3 or more p-waves of different morphologies, narrow QRS complexes, variable P-R and R-R intervals = dx? Causes?

A

Multiform atrial tachycardia

Causes = hypoxia, COPD, hypokalemia, hypomagnesemia, CAD

519
Q

Splenic vein thrombosis can occur as a complication of what in alcoholics?

A

Chronic recurrent pancreatitis causing anemia, thombocytopenia, and leukopenia - can have gastric varices without esophageal varicose;

520
Q

Trastuzumab-related cardiotoxicity pr/w w/ asymptomatic decline in LV EF and sometimes overt HF - is it reversible?

A

Yes; decline is unrelated to cumulative effects

521
Q

Patients p/w hypogonadism and low gonadotropin levels and elevated alpha-subunit, vision changes, mild elevation of prolactin = ?

A

Nonfunctioning pituitary adenoma - treated with trans-sphenoidal sx

522
Q

Transurethral resection of the prostate (TURP) associated w/ what electrolye complication?

A

Hyponatremia 2/2 20-30 L of isosmotic flushing flushing solns

523
Q

Cherry red flushing, later occurring cyanosis, AMS, seizures, coma, arrhythmias, pulmonary edema, abd pain, nausea, vomiting, metabolic acidosis w/ nitroprusside = dx? Trmt?

A

Cyanide toxicity - treat with sodium thiosulfate

524
Q

After suspected HIT and heparin stopped, when do you restart Warfarin?

A

When PLT recover to > 150,000 - it’s usually restarted with a non-heparin anticoagulant

525
Q

Painless blisters on dorsum of hand , increased fragility of the skin, facial hypertrichosis, and hyper pigmentation = dx?

A

Porphyria cutanea tarda - deficiency of uroporphyrinogen decarboxylase

Phlebotomy or hydroxychlorowuine may provide relief with interferon alpha in those affected by Hep C.

526
Q

Patient’s with rhinitis can take what before scuba diving?

A

Non-sedating decongestants (pseudoephedrine) - can reduce incidence of ear and sinus barotrauma by 75%

527
Q

Major problem that leads to difficulties in finding cross-matched blood in pts with a hx of multiple transfusions = ?>

A

Alloantibodies

528
Q

Different b/w first degree and second degree block? Difference b/w second degree block? What makes them worse/better?

A

First degree = prolonged PR interval with no dropped beats.

Second degree (type I Mobitz) = progressive PR interval with subsequent dropped beat (exercise/atropine makes better; carotid massage makes worse) VS. Type II Mobitz = maintained PR interval but intermittent dropped beats (exercise/atropine makes worse and carotid massage improves)

529
Q

What heart block patients get permanent pacemakers?

A

Symptomatic Second Degree Type I Mobitz and Type II Mopbitz being asymptomatic/symptomatic; can use temporary pacer

530
Q

Patients w/ refractory hypoglycemia 2/2 possible sulfonourea trmt?

A

Octreotide because somatostatin analogue

531
Q

Treatment for hyperkalemia causing EKG changes?

A

Calcium Gluconate - see peaked t-waves, loss of p wave, widened QRS w/ sine wave

532
Q

Jaundice, fever, RUQ pain = dx? Mgmt?

A

Acute Cholangitis - blood cultures and abx (gent+amp or levofloxacin or imipenem), fluids! Then ERCP afterwards

533
Q

Loss of pupillary reaction, vertical gaze paralysis, loss of optokinetic nystagmus, and ataxia = dx?

A

Parinaud’s syndrome 2/2 Pineal tumor. Can have HA from obstructive hydrocephalus. Can sometimes secrete B-hcg causing precocious puberty

534
Q

Abx to prevent endocarditis is necessary when?

A

Dental procedures, pulm procedures w/ bx, GI/GU procedure w/ concomittant infection, procedures on infected skin or MSK

535
Q

With someone experience urinary incontinence, ataxia, and loss of cognitive function, what’s the first test to determine mgmt?

A

LP puncture - if they get better, can proceed with Ventriculoperitoneal shunt 2/2 normal pressure hydrocephalus

536
Q

Physiologic jaundice occurs how long after birth?

A

24 hrs

537
Q

MC deficiency causing hemolysis w/ unconjugated hyperbili in newborn from africa/med/asia?

A

G6PD - usually asymptomatic when young

Thalassemias and SCD usually manifest later because baby is dependent on HgbF

538
Q

Criteria for delayed puberty in male? Tests?

A

No teste enlargement by 14yrs - MCC = constitutional delay

Need bone age via L wrist X-ray

539
Q

PEA/asystole trmt=?

A

CPR and vasopressor therapy; search for reversible causes: Hypoxia/Hypothermia/Hydrogen Ions/Hypovolemia/Hypo-Hyperkalemia

TPTHX/Tamponade/Thrombosis/Trauma/Toxins

540
Q

Lung Associations =
1) Small Cell-

2) Asbestosis
3) Mesothelioma -
4) Large Cell
5) Squamous Cell Cancer

A

1) SIADH
2) Fibrosis- shipyard workers, chest tightness, wheezing
3) Linked to asbestos; nodular thickening of pleura
4) Large cell - large peripheral mass
5) Cavitary lesion on bronchus - hypercalcemia 2/2 PTHrP

541
Q

Thickening of outer bone cortex, bowing, sclerosis, increased uptake on Technetium bone scan = dx and trmt?>

A

Pagets - treat if bone pain, weight-bearing bones affected, neuro compromise, hypercalcemia, hypercalciuria, and CHF w/ bisphosphonates

542
Q

Malignancy risk in celiac patients?

A

Intestinal T cell lymphoma - affects Jejunum commonly, nodular/ulcerative; surgery and chemo - high relapse rate

543
Q

Fever, facial swelling, maxillary pain, tenderness, nasal discharge thats foul smelling, opthalmoplegia, and HA w/ black discoloration in DM pt = dx? trmt?

A

Mucormycosis 2/2 to fungal infection from Zygomycetes - biopsy to confirm - debride and give amphotericin B

544
Q

OCD trmt?

A

CBT special for desensitization and serotonergic antidepressants (SSRI’s and Clomipramine, a TCA)

545
Q

When to give IV IG to ITP patient?

A

Plt

546
Q

Trmt of acute dystonic reactions?

A

Diphenhydramine or Anticholinergic rx’s (benztropine or trihexyphenidyl)

547
Q

BP mgmt in gouty pts?

A

ARB’s

548
Q

Abx for suspected ped sepsis based on age?

A

28 = Ceftriaxone or Cefotaxime +/- Vanc (if meningeal involvement suspected) - Step PNA/Neisseria

549
Q

Initial test for suspected exogenous hyperthyroidism = ?

A

24-hr radioactive iodine uptake test

550
Q

Best PE test for achilles tendon rupture?

A

Thompson test = squeeze gastroch

551
Q

Myaesthenia gravis associationg w/ what thyroid problem?

A

Thymoma